Sie sind auf Seite 1von 457
SP ig (OLY a —— Wt atsies) tne) SSO SW Ny WASTES 5 VR a arabeerad Published by : FIREWALL MEDIA (An Imprint of Laxmi Publications Pvt. Ltd.) 113, Golden House, Daryaganj, New Delhi-110002 Phone : 011-43 53 25 00 Fag : 011-43 53 25 28 www.laxmipublications.com info@laxmipublications.com ISBN : 81-7008-939-5 Copyright © 2007 by Firewall Media (An. imprint of Laxmi Publications Put. Ltd.). All rights reserved. No part of this publication may be reproduced, stored in a retrieval system, or transmitted in any form or by any means, electronic, mechanical, photocopying, recording or otherwise without the prior written permission of the publisher. Price : Rs, 250.00 Only. Tenth Edition : 2006 Offices: India USA © Bangalore (Phone : 080-26 61 15 61) © Boston © Chennai (Phone : 044-24 34 47 26) 11, Leavitt Street, Hingham, © Cochin (Phone : 0484-239 70 04) MA 02043, USA © Guwahati (Phones : 0361-254 36 69, 251 38 81) Phone : 781-740-4487 @ Hyderabad (Phone : 040-24 75.02 47) » Jalandhar City (Phone : 0181-222 1272) © Kolkata (Phones : 033-22 27 37 73, 22.27 52.47) © Lucknow (Phone : 0522-220 95 78) Mumbai (Phones : 022-24 91 54 15, 24 92 78 69) Ranchi (Phone : 0651-230 77 64) MGA.6706-250-G, DIFF. EQUATIONS C—13019/06/08. Typeset at : Goswami Printers, Delhi. Printed at : Sanjeev Offset Press, New Delhi. CONTENTS Chapters Pages List of Important Formulae wa _(i)}(wi) 1,__Differential Equations and their Formation an 1 2.__ Solution of Differential Equations of the First Order and First Degree 21 3.__Linear Equations with Constant Co-efficients a 170 4. Application to Geometry and Mechanics es 269 5.__Homogeneous Linear Equations oe 286 “6._ Trajectories . 314 7, Equations of the First Order but not of the First Degree so 326 8.__Linear Equations of Second Order 356 9. Simultaneous Differential Equations se 400 10. Series Solution of Differential Equations as 429 11. Legendre’s Equation os 512 12.__Bessel’s Equation se 533 GET 100 OUT OF 100 MARKS The book has been written with a view to guide the students in the examination. It ensures 100% marks ‘The student is advised to take this book only as « help book. He should solve himself as many questions as possible. = Special Features : 1. Solutions to all types of important questions have been given. 2. Questions from up-to-date papers of various Universities. 8, The book is useful to bright as well as weak students alike. 4. Approach to the subject matter is systematic. 5. The Book serves triple purpose : ‘Text Book—Help Book—Solved University Papers List of Important Formulae (TO BE LEARNT BY HEART] L VARIABLES SEPARABLE To solve x = XY, where X is a function of x only and ¥ is a function of y only. Bring all the terms of x and dx on one side, all the terms of y and dy on the other. Integrate both sides and add an abritrary constant on one side. 2, HOMOGENEOUS EQUATION dy fsy) dz f(x,y) tions of the same degree in x and y. do dy Puty = ox, sothat ST su +x 2 To solve the equation where f(z, 9), f,(, y) are homogeneous func- Substitute the values of y and a in the given equation. Separate the variables v and x. Integrate both sides and add an arbitrary constant on one side. Substitute back the value of v (- 2) . 8. EQUATIONS REDUCIBLE TO HOMOGENEOUS axtbyte ab = aerbiyre ‘When . Putx=X+hy=Yok Equate the constant terms in the numerator and denominator of R.H.S, to zero and find i dy To solve The equation is now homogeneous in X and Y. Put ¥ = oX and proceed as in the ease of homogeneous equations. Substitute back the values of v, X,Y, A.and @ (ii) 8‘ dy __axtby+e_ (ii) To solve the equation 7 = Gra biyse' dy Put the values of ax + by and ©” in the given equation, Separate the variables z and x and proceed as in Variables Separable. Substitute back the value of z. 4, LINEAR EQUATION (i) To solve & + Py = Q, where P, Q are functions of x only. Make the co-efficient of # unity, if not so already. Find. LF, = e!?* and remember that cl! = fix) ‘The solution is y . (LF.) =/ Q(LF.) dr +c. (i) To solve $= + Px = @, where P, @ are functions of y only. dx Make the co-efficient of 7, unity, if not so already. Find ifpseh’ ‘The solution is x. (LF.) = J Q(LF.) dy +c. 5. EQUATIONS REDUCIBLE TO LINEAR (To eotve + Py = Qy", where P, Q are functions of x only. Divide throughout by y" to make the R.H.S, a function of x only. Put yhrez The equation is now linear in z. Make the co-efficient of & unity, if not so already. Find LF. =e! Pe The solution is 2. (LF) =/Q.0F) de+e Substitute back the value of z. (ii) To solve the equation f(y) x +Pily) = Q, where P, Q are functions of x only. Put fly) = z, then f(y). & & ae de. Given equation becomes < + Pz = Q which is linear in z and can be solved. Giiy 6. EXACT DIFFERENTIAL EQUATION aM ol The equation Mds + Ndy = is exact Se = 2. Thesolution is f —Mdx+ J (terms in N not containing x) dy =. -yeonstant 7. INTEGRATING FACTORS (i) If the equation Mdx + Ndy = 0 is homogencous in x and y, then mw is an integrating factor I Mx +Ny #0 ‘The method is used when Mx + Ny consists of only one term. (ii) If the equation Mdx + Ndy = 0 is of the form Pay y du + fifa y) x dy = 0, then i ‘Mx -Ny -Ny (iii) Ifin the equation Md + Ndy = 0, aM _ oN a is a function of x only =/ix) (say) is an integrating factor | Mx —Ny #0 then e!/'*"¢ is an integrating factor. (jv) If in the equation Mdx + Ndy = 0, aN aM oY is a function of y only = fly) (say) then e!/ "4 is an integrating factor. 8. TO SOLVE THE EQUATION ay. avy atty Ser oe oe = where a,, ay, «.., a, are constants. Write the given equation in symbolic form a D* =p" nm Thus (DP +a,D™1 4 a,D"2 4... 4a,y =0 Write down the auxiliary equation (A.E.) by equating to zero the symbolic co-efficient ofy Thus D' +a, D144,D"%4.... 44, =0 Solve if for D as if it were an ordinary algebraic quantity. (iv) From the roots of the A-E., write down the corresponding part of the complete solution as follows : Roots of AE. Corresponding part of C.S. |. One real root m, ce . Two real and different roots m,, m,, ce™ +ege** |. Two real and equal roots m,, m,. (ey #e00™" |. Three real and equal roots m.,, m,, ™m, Cc, tg + e716" . One pair of complex roots «+ if. ce (e, cos x +e, sin fix) ‘Two pairs of complex and o [(e, + e,¢) cos fr + (cy + eye) sin Be) equal roots a + i, a iB. Complete solution is y = corresponding part of C.S. 9. RULE TO SOLVE THE EQUATION a™ty tay Gore ton tay where a, a... 8, are constants and X is a function of x. and Write down the given equation in the symbolic form (* +a,D*+0,D"* + Auxiliary equation is D* + a,D*! +a,D"* +...... Solve it for D as if it were an ordinary algebraic quantity. From the roots of the auxiliary equation, write down the complementary function (C.F.) 1 1 Ple pea p sabes .cta, *= 70D) * 1 1 Case l Fi) = Fa) where fla) #0 1 1 Case IL Fon, sin as = Fg) tin ar where fl-a2) #0 1 1 FD®) = Fa?) OH where ft- a2) #0 1 Case IIL. Fp, =" where m is a positive integer. ‘Write AUD) in ascending powers of D and make the first term unity, if not so already. 1 1 1 Let FD 7" * olism "2g tO Expand it by Binomial Theorem, remembering that (+ert=1-#+P-8+.. (at sletePet en... (4 ty? 21-24 3tt- 4 + (1-07? = 14284 87+ 4P +o, 0) The expansion is to be carried out upto D” only. Operate on x” with each term of the expansion. Case Tv. Fi “Ara * 1 Lyd (a a Case V. Fi *V=* fay) V+ ap LF) Y=" FD where V is a function of x. 10. CASES OF FAILURE If by using the above rules for finding the P.I., we get zero in the denominator, then (fF a a Fn ay" when fla) = 0 @ 1. 1 . —— sin ax =x, 4 ___ sin ax when fl- a2) =0 fo?) fp? f Son Fah TE hoses when f-a2)=0 ro) aD he, x. —————! __ 4" or sin ax or cos ax whatever the case is, diff, coeffi. of denom. w.r.t.D If we again get zero in the denominator, repeat the above process. lL. z = integration. 12, es cosxtisinx so that cos x is the real part of e* and sin x is the imaginary part of e*. 18, RULE TO SOLVE THE EQUATION a’ a"! no BY ayn Ty where ay, &,, My, ....., 8, are constants and X is a function of x. {Homogencous Linear Equation of the nth order} Putx =e" so that z = log x and sae sare dd Dak ae de a ‘Then Pag =DD-) 23S, = DD - 1D - 2) and so on. (wi ‘The given equation reduces to (Diy = Z, a function of z. Solve it and replace z by log x. 14, RULE TO SOLVE THE EQUATION where By 81) ayy. &, are constants and X is a function of x. Put a + bx =e* so that z = log (a + bx) and De & ,thenta+b2) 2 =o, o (a + bx? a =5°DD-1) e (a+ beh Te = PDD - 1D -2) - ‘The given equation reduces to /(D)y = Z, a function of z. Solve it and replace z by log (a + br). 1 Differential Equations and Their Formation Definitions 1. Differential Equation. A differential equation is an equation which involves differential co-efficients or differentials. ees tog, dy = 008 eds, dy dy . ae 4 Gy 7 lay = be + sine 2, 2. Ordinary Differential Equation. An ordinary differential equation is that which involves only one independent variable and differential co-efficients w.r-t. it. The above examples are of this type. 3. Order of a Differential Equation. The order of a differential equation is the order of the highest differential co-efficient which appears in it. eg,, the differential equation 2. a + dy = e* is of second order. 4. Degree of a Differential Equation. The degree of a differential equation is the degree of the highest differential co-efficient which appears in it, when the differential co- efficients are free from radicals and fractional powers. 24,\% 2, eg. the differential equation [+(2)] =a? (3) is of degree 3, the degree of = Example. Find the order and degree of the differential equation = . ay wy of. fay? ay yen ea r+ #) i) 1+(2) sac J Sol. (i) Sea iit : o yor Bea fio(8) Squaring [to get rid of radicals] bra) -«[(8] which is of the first order and second degree. [highest degree of dy /dx is 2] 2 GOLDEN DIFFERENTIAL EQUATIONS 282 dy’ 2. (ii) [1-(2) ] za Squaring [to get rid of radicals] a ay [H(gh J . “Sf which is of the second order and second degree. [: order of highest differential co- -oficiont 2 Jis 2and highest 2. degree of 3 , the highest differential co-efficient, is | Solution of a Differential Equation. A solution of a differential equation is a func- tional relation between the variables involved which satisfies the equation. General Solution. The solution of a differential equation in which the number of arbi- trary constants is equal to the order of the differential equation is called the general solution [or complete solution or complete integral or complete primitive]. Particular Solution. If particular values are given to the arbitrary constants in the general solution, then the solution so obtained is called a particular solution. Singular Solution. A solution which does not contain any arbitrary constant and also, is not obtainable from the general solution by giving particular values to the arbitrary con- stants, is called a Singular Solution. Example 1. Show that 2 (i) 2 +dy =O isa solution of (2) + Gii)y=— S+disa solution of > ay, & er) (DD The given function is x? + 4y =0 ol2) x wea Boo 2 Sol. (i) The given differential equation is (2) +x Differentiating (2) w. x = Ses dx 2 Substituting for y and * from (2) and (8) in (1), we get (ala (-4)-0 DIFFERENTIAL EQUATIONS AND THEIR FORMATION 3 or xt 2st ax? =O or which is true, <. Equation (2) is a solution of (1). [Since this solution is free from arbitrary constants, it is a particular solution.] 7 von differenti on f2 42 Gi) The given differential equation is [3 + =. 9 AD The given function is y= £ +d (2) . cos ad c Differentiating (2) wrt. x Ga-3 (3) d?y Differentiating again w.r.t. x 3.5 (4) a? x 2 Substituting for & and a from (3) and (4) in (1), we get. de 2 e 2c _ 2 Bei(a)-0 « BR which is true. =. Equation (2) is a solution of (1). [The given diff. eq. is of second order and the solution (2) contains two arbitrary con- stants viz. ¢ and d, Hence this solution is the general solution.) Example 2. Verify that y = (x - x) log cx satisfies the differential equation (3 —x)y' — (8x? - Dy =x 2942 for all values of ¢. Sol. The given differential equation is G3 — xy — Be? — My = 25-23 ex (1) ‘The given function is y = (23 =x) log cx Differentiating wr-t. x, yett—x). 4 c+ taet—wloger or ¥ = (2-1) + (8x? - 1) log er Substituting the values of y and y’ in (1), we have (a3 — x)? - 1) + (8x? - 1) log ex) — (8x? - 1x? — x) log ox axS- Ore or Bax Xet— 1) + (24 — 24822 — 1) log ex — (82? — 1) — x) log ex sabre or dae eso ew which is identically true. Since the given function satisfies (1), it is a solution of (1). 4 GOLDEN DIFFERENTIAL EQUATIONS Example 3. Show that y = —(1 + x) is a solution of the differential equation (y — x) dy - Gf -2)dz=0. Sol. The given differential equation is 0 -x) dy 07-24) dx=0 D ‘The given function is y =— (14x) 90 that dy =-de Substituting the values of y and dy in (1), we have (-1-x-x)(-dx)-[(1 +2? -2x7)dr=0 or (1 + 2x) de - (1+ 2x) de=0 which is identically true. Since the given function satisfies (1), it is a solution of (1). Example 4. Show that the function y = e* (A + Bx) is a solution of the differential equation £2 6 Ps oye0 A) The given function is yee (A+ Br) ol2) Differentiating w.r-t. x, & =e* B+ 3e™ (A + Br) 2 = Bel + ay (8) (Using (2)) Differentiating again w.r.t. x, dy Er = SB 4S ” or a. [z - sy] +32 [Using (9) x ae 2, or oF 6S +oys0 which is the same as (1). Hence, the given function is a solution of (1). Example 5. Show that the function y = Ax + 3 is a solution of the differential equation aa -y=0 wf) (DIFFERENTIAL EQUATIONS AND THEIR FORMATION 5 ., a e. B ‘The given function is y= Ax+ 50 that Yea-5 ond = x ae 2 Substituting the values of y, 2 and 3 in (1), we have o(B)oa(t Bed) or By ae-B-a-B 0 * x x which is identically true. Hence the given function is a solution of (1). Example 6. Show that the function y = ae™ + be* ia a solution of the differential equation dy dy BR ee Sol. The given differential equation is d’y dy Ek =0 (1) ‘The given function is y sae™ + be* oY ane = be* ay ast so that de 720% — bet and TF = dae + be* Substituting the values of y, e end © < in (1), we have (dae* + be*) — (2ae* — be 2 (ae™ + be*) = 0 or (4a — 2a ~ 2a) &* + (b + b-2be* = 0 which is identically true. Hence the given function is a solution of (1). Example 7. Show that the function y = be‘ + ce* is a solution of the differential equation a? ds a 9 42y=0. set Please try yourself. FORMATION OF DIFFERENTIAL EQUATIONS Rule to form the differential equation from a given equation is x and y, con- taining arbitrary constants. 1. Write down the given equation. 2, Differentiate w.rt, x successively as many times as the number of arbitrary constants. 8. Eliminate the arbitrary constants from the equations of the above two steps. The resulting equation is the required differential equation. | GOLDEN DIFFERENTIAL EQUATIONS Example 1. Find the differential equation of all straight lines passing through the origin. Sol. Equation of any straight line through the origin is . yams AD 'm being the parameter. Differentiating wrt. x, gy - an™ wi) Now we have to eliminate m between (i) and (ii) . a yy. dy From (i), m=2 Prom iii), F = > x & =y which is the required differential equation. Example 2. Find the differential equation of all non-vertical lines in the xy-plane. Sol. The equation of any non-vertical line in the xy-plane is yamrte lB) It contains two arbitrary constants viz. m and ¢ Differentiating (i) w.r-t. x # =m . iat i d’y Differentiating again © 3 =0 which is the reqd. diff. equation. Example 8. Obiain a differential equation that should be satisfied by the family of concentric circles x* + y* = a”, lor f fe rg or Sol. x? + y? = o?, Differentiating wart, x, 22 +27 Boo {a is eliminated] x+y s = Owhich is the required differential equation. Example 4. Find the differential equation of the system of circles touching the y-axis at Bol. Any circle which touches the y-axis at the origin must have its centre on the x-axis. Equation of such a circle is x+y? 4 2ge=0 i) Differentiating (i) w.r.t. ‘x’ d+ ay. La rg=0 oléi) ‘Now we have to eliminate g between (i) and (ii) From (ii), Iv ofa) tite dy, Pesta yt Day & +22_y? = 0 which is the required differential equation. DIFFERENTIAL EQUATIONS. AND THEIR FORMATION 7 Example 5. Find the differential equation of all circles touching a given straight line at a given point. Sol. Take the given point as the origin and the given straight line as the axis of y. Now proceed as in Example 4. Example 6, Find the differential equation of the family of curves (x— hy? + (y - kJ? = r?, where h and k are arbitrary constants. Sol. The equation of all circles of radius r is (AP + y-kP =r? where A and k, the co-ordinates of the centre are arbitrary. Differentiating both sides of (i) w.r.t. ‘x’ x—h)+2y-h). or @-h+y-h). Diforentating agnin wt From (iii), y-k ah oad re dy) |dy sd?y = From (i), shag @ «[r+(¥)] a Substituting the values of x —h and y—k in (i) cae ote (cs) (a) «(Ger - f(a) Ca) “ teh] a -«(ca) 7 [ay] nC which is the required differential equation. Another Form. Find the differential equation of all circles of radius 'r. wl) =i) (iii) 8 GOLDEN DIFFERENTIAL EQUATIONS Example 7. Find the differential equation of all circles in the xy-plane. Sol. The general equation of a circle in the ay-plane is ey? + Oger + Oy te = where g, f,c are arbitrary constants which are to be eliminated. Differentiating ()) w.r.t. ‘x’ Bet Qyy, + Be + Br = 0 or xtoyy tet fy =0 sulGi) {c is eliminated] wel) Differentiating (ii), wart, Ltoyg tyy? +f, = 0 iti) lg is eliminated) Differentiating (iii) w.r.t. ‘x’ Yad'g + V9 + By p¥e + fy = 0 ww) [to eliminate f) Multiplying (ii) by yy and (iv) by yy Yat Ways + Hrs + Hayy =O wv) Wills + V9? + Bye? + roy, =0 Avi) Subtracting (vi) from (v) Ya +¥¥a~ By? = 0 or (1. +y,75~ 3y,y.? =0 which is the required differential equation. Example 8. (a) Find the differential equation of all circles which pass through the origin and whose centres are on the x-axis. (b) Find the differential equation of all circles which pass through the origin and whose centres lie on the y-axis. Sol. (a) The equation of any cirele passing through the origin and whose centre is on the maxisis x? +524 2gr =0 Now proceed as in Example 4. (6) Please try yourself. 4 ([Blint. x2 + y? + 2f7 = 0) [Ans tat -¥*) bey -0] Example 9. What is the order of the differential equation whose solution is the cirele (e-0) + ¥ = 02, where ais an arbitrary constant ? Sol. The given equation is (x - 0)? +9? = a? or x4y?-20r=0 D) Differentiating (1) w.r.t. x, ox 42y® ~20=0 d) Substituting this value of a in (1), we get seyt-2e(e+r 2) -0 DIFFERENTIAL EQUATIONS AND THEIR FORMATION 9 dy dy or yaat—2y Pad or tay Fe aty2=0 which is the differential equation of the family of circles (1) and it is of first order. Example 10. Show that the differential equation of the family of circles of fixed radius r with centres on y-axis is - ey - at-r9(B) +20, Sol. Let the center be (0, #), then the equation of circle is 224+Q-bP ert wf) where & is an arbitrary constant. Differentiating w.r-t. x, 2x + 2y — hy’ = O where y = SE = yok Substituting the value of (y — &) in (1), we have 2 B+ ter or (x2 ry? +2730 2 or GP — 79) (2) + x* =0 which is the reqd. differential equation. Example 11. Find the differential equation of all parabolas whose axes are parallel to y-axis. Sol. The equation of any parabola whose axis is parallel to y-axis is . (x-h}? = daly —k) where 4, k, a are three arbitrary constants. Differentiating (1) wrt.x = h)= da s d’y Differentiating again Q=4a a . dby Differentiating again O=4a 5 dy sch i ‘ . oo es 0 which is the reqd. diff. equation. Example 12. Find the differential equation of all parabolas with latus rectum “4a” and whose axes are parallel to x-axis. Sol, The equation of any parabola with latus rectum 4a and axis parallel to x-axis is (y—k)? = dale ~A) o() where h, k are two arbitrary constants, Differentiating (1) w.rt.x 2y—h) # ota y-m % = (2) 10 GOLDEN DIFFERENTIAL EQUATIONS. 2 2 Differentiating again w.r.t. x (y—&) 3 + (4) =0 248) 2a From (2), yk se dx Substituting the value of (y — k) in (3) 2a dty (2) ay ( y - Get e) -° Gr +(G) =e dz which is the reqd. diff, equation. Example 13. Find the differential equation of ail conics whose axes coincide with the axes of co-ordinates. (Indore, 1997 ; Gauhati, 1996) Sol. The equation of any conic whose axes coincide with the axes of co-ordinates is Ax? + By?= 1 sD) where A, B are two arbitrary constants, Differentiating (1) wnt. x 2Ax + 2By wy =0 = Ax 4 By 2 =0 (2) Differentiating again w.r.t. x ay dy asn(2) +By Sp e0 . From (2), * = -2 dy dy From (3), = -[{ | 2 " x dx which is the reqd. differential equation. Example 14, Find the differential equation of all ellipses centred at the origin. Sol. The equation of any ellipse centred at the origin is Ax? + By?=1(A > 0, B> 0) Now proceed as in Example 13. Example 15. Prove that the differential equation of the family of parabolas y? = dax is ey br Fe 720. Sol. Please try yourself. DIFFERENTIAL EQUATIONS AND THEIR FORMATION WW Example 16, Find the differential equation of all the circles in the first quadrant which touch the co-ordinate axes, Sol. The equation of all the circles in the first quadrant which touch the co-ordinate axes is (r-a? +(y-a? =a? ww) or x2 4y2-2ax—2ay +a?=0 Differentiating wart. x, we got " = 2a - 2ay’ = 0, where = 2 or x+y salty) xtyy o= thy Substituting the value of a in (1), we get nt ne 2 Lat" _zta') (sty" (52) +l 87) EF) axe or or Gy! yy’ + ya = + 9’ or 97 =P (aay ales yy? or G-yP [1+ ¥7] = +57"? af. (4y le ayy . oof oor] which is the required differential equation. Example 17. Show that the differential equation that represents all parabolas having their axis of symmetry coincident with the axis of x is 392 +977 =0. Sol. The equation of any parabola having its axis of symmetry coincident with the axis of xis y= 4a (x-h) wa) where @ and h are arbitrary. Differentiating w.r.t. x, Qyy, = 4a or yy; = 2a Differentiating again w.r.t. x, wy ty' =0 which is the required differential equation. Example 18. Find the differential equation of a family of central conics represented by ey a 4h * aR Sol. Given equation is = 1, where 2. isa parameter. ww 1) 12 GOLDEN DIFFERENTIAL EQUATIONS Differentiating w.r-t. x, wo get ax ayy’ dy =0, where y= ek Bn WN ae or xb? 42) + yy? +0) = 0 or Mo + yy’) = - (Bx + ayy’) Bieta*yy’ or oS rey atadeat- Petey _a* be xtyy’ and =- 2 xt yy Substituting these values of (a? + A) and (6 + A) in (1), we get Baty) yaetw) | (a? -B8)x (a? - bby’ or wie + yy) - M4) gag or PY #xyy? = x9 -y°¥ = (a? = 67 or ayy? + (x2 y? a8 + by’ ay = 0 2 ow (2) seeps Sano which is the required differential equation. Example 19, Form the differential equation corresponding to y= mfa? ~x4) by eliminating m and a. Sol. The given equation is y? = m(a? - x) AD Differentiating w.r.t. x, we get 2yy’ = = Qe Differentiating again w.t.t. x, we get 2) From (2), y= xy" #9) or syn” + ay? — yy a ra o offs) 1H which is the required differential equation. Example 20. Find the differential equation of the family of curves represented by oy +cPh =x, Sol. The given equation is c(y+c)*=25 AD Differentiating w.r-t. x, we get ely + e)y” = Bx? 2) DIFFERENTIAL EQUATIONS AND THEIR FORMATION 13 sas yee x Dividing) by, Fag wow wey or yeengm’ 2 em gay-y Substituting the value of c in (2), we get 2(2a9' -»}(2 wy ax? or Ascy? (Dey? — By) = 272 or Bay — Ly? = 27x or x (By? = 27) = 12yy? 3 ay o -|o(2)-27]- (2) which is the required differential equation. Example 21. Find the differential equation corresponding to the family of curves y = fx ~ c, where e is an arbitrary constant. Sol. The given equation is yactx—ci? AD) Differentiating w.r.t. x, y = 2elx-e) «f2) Dividing (D by (2), BaA5f or cure Substituting the value of ¢ in (2), we get (2:-42). or tay by! —2y) a « (2) = (--») which is the required differential equation. Example 22, Find the differential equation of the family of curves y = Ae™ + Be*, (Punjab, 1996) Sol. The given equation is y = Ae* + Be®* AD) It has two arbitrary constants A and B. Differentiating, a = DAet — 2Be-2 dy Differentiating again, pad = Ae + 4Be2e = 4(Ae™ + Be*) =4y [Using ()] or 3 — dy = 0 which is the required differential equation. Example 23. Find the differential equation of the family of curves y= Acos (x2) + B sin (x*). irs GOLDEN DIFFERENTIAL EQUATIONS Sol. The given equation is y = Acos x? + B sin x? A) Differentiating, AC sin x). 204 Bloons?) 28 = 2x Asin 2? + B cos 22] 2) Differentiating again. 2, SF a2 [-Asingt + Bosal] + 2e[-A (cons!) 2 + B (sin 32). 22] 2 ® ; ; + gg ~ APTA cos 2? + B sin x2] (Using (2)] 4B o eB-Bsuryn0 which is the required differential equation. Example 24. Find the differential equation of the family of curves y= Ae™ + Be~*, for different values of A and B. Sol. The given equation is y = Ar + Be old) Differentiating war.t. x = 2Ae* — SBe“t alti) Differentiating again w.r.t. ee ey = 4Ae™ + 9Be“* .-Aiii) Adding (ii) and (iii), fy Donets Bee <6) > offi) or 8 ® geo which is the reqd. imental equation. Example 26. Find the differential equation of the family of curves y = Ae + Be®, where A, Bare arbitrary constants. Sol. The given equation is y = Ae™ + Be A) Differentiating w.r.t, x, we get y= SAe™ + 5Be™ 2) Differentiating again w.r.t. x, we get yf = 9Ac™ + 25Be™ (8) [Now we find Ae* and Be™ from (2) and (3)] ‘Multiplying (2) by 5 and subtracting (3) from the result, by’ =y" = 6Ae* or Ads Ly -¥) Multiplying (2) by 3 and subtracting the result from (3), ¥’ ~3y = 10Be* or Be = "= 39) DIFFERENTIAL EQUATIONS AND THEIR FORMATION 15 Substituting the values of Ae and Be® in (1), we get 1 ny, ” y= gy’ -¥)+ 75 0-8 or By = 25y’ — By” + By” — By’ or Qy"—16y'430y=0 ory" By" + Ly =0 or #- 82+ 15y- 0 which is the reqd. differential equation. Example 26, Find the differential equation of the family of curves y=ae™ + be* Sol. Please try yourself. [ame ay -52 +6y=0 Example 27. Find the differential equation of the family of curves y = e™, where m is an arbitrary constant. Sol. The given equation is y =e" AD) Taking logarithms, log y = mx Differentiating w.r.t. x, we get 1 dy y de Substituting the value of m in (1), we get logy= 5% or x vlog y=0 which is the required differential equation. Example 28. Find the differential equation which has e+ 2aze’ +a? = 0 for its complete primitive. Sol. The given equation is e” + 2ax &’ + a? =0 It is a one parameter family of curves. Differentiating (i) war.t. x 2 y+ 2ae? + axe”. yy = 0 = ye +a(1 +x) =0 (Dividing by 2e*] = ae 1+ Putting this value of a in (i), we get ae" wer eo — Bee 7 + SES =0 ltay, * Gey? or 2% (1 + sy) — Day, (1 + 9y,) +y,2 2 = 0 or 14 2ay, + a2y,2— Bay, ey? 9,2 = 0 or Lary2+y2=0 or (1-2%)7,24+150 16 GOLDEN DIFFERENTIAL EQUATIONS 2 a a(S 1-8 which is the reqd. differential equation. Example 29. Find the differential equation corresponding to y= ae™ + be™ + ce where a, b, c are arbitrary constants. Sol. The given equation is y= ae" + be + ce Differentiating w.r.t. x 9, = Get + 2be™* — Boe = (aet + be® + ce-S) + be® — doe = yy=y + be™*— doe Differentiating again Yq = Wy + Be™ + 12ce-% = yy + UAbe™ — doe“) + 20ce-* 5, +20, -9) + 20ce* => Yq~ By, + By = 20c0-* Differentiating again w-r.t. x ¥3— Byq + By, = 60ce — 3(20ce-*) ~ 8-39, + 2y) or ¥g~ Ty, + Gy =O which is the reqd. differential equation. Example 90. Find the differential equation of the family of curves y =A cos mx + B sin mx where m is fixed and A, B are arbitrary constants. Sol. The given equation is ym Aces nie + Besin ite Differentiating w.rt. x 2 mA sin ms + mB cos me Differentiating again dy — mA cos mx — m®B sin mr dx* =— m2 (A cos mx + B sin mx) = — my d?y dx? which is the reqd. differential equation. or +my =0 wn) wuB) (Using (D1 [Using (2)) +3) [Using (8)] A) [Using (2)) 81. Form a differential equation of which y = e* (A cos 2x + B sin 2x) isa Example solution . (A and B being arbitrary constants). Sol. The given equation is y = e* (A cos 2x + B sin 2r) Differentiating wrt. 2 = e* (A cos 2x + B sin 2r) + e* (~ 2A sin 2x + 2B cos 2x) ld) DIFFERENTIAL EQUATIONS AND THEIR FORMATION 0 or SY oy +e 2A sin De + 2B cos 2x) wwlié) Ie of @ Differentiating again w.r.t. x d’y dy , . Qi ds +e (— 2A sin 2x + 2B cos 2x) + e* (— 4A cos 2x -- 4B sin 2x) dy dy . Ey) — ter (8 cos 20 + B sin 2s) ae le of i) a2 2 yay ls of ® By 9 WY pye me 2a, rea which is the required differential equation. Example 82. Find the differential equation fram the relation y = e* (A cos x + B sin x), where A and B are arbitrary constants. (Punjab, 1996) ay ia Sol. Please try yourself. Ans, Fy -2 G+ By=0 Example 33. Find the differential equation from the relation y = a sin x +B cos x + x sin x, where a and b are arbitrary constants. Sol. The given equation is y= asinx + bcos x +xsinx sulé) Differentiating w.r.t. ‘x’ dy Fe 2 a005x—b sina + sin x +x 005.x Differentiating again w-r.t. x, 2 Sten sin x—b cos x + cos x + cos x—x sinx =-(asinx +b cosx+xsinx)+2 cosx =-y+2eosx ly of 2 * 3 +9 =2 cos is the required differential equation. Example 34. Find the differential equation of the family of curves xy = Ae? + Be* + x? for different values of A and B. Sol. The given equation is ay = Ae“ + Be* +x” on(é) Differentiating wart. x yer & = Act = Be* + 2x Differentiating again w.r.t. x oa ay det de ** git Ae + Ber+2 18 GOLDEN DIFFERENTIAL EQUATIONS or 28s hey 242 les of 2% +2 amystea is the required differential equation. Example 35. Find the differential equation which has y = a cos (mx + b} for its complete integral, a and b being arbitrary constants. Bol. The given equation is y =a coe (mx +8) caf) or Differentiating w.r.t. x s = — me sin (mx + b) 2 Differentiating again - mia cos (mx +b) =— my {Using (1)] or £ +my=0 which is the reqd. differential equation. Example 36, Form a differential equation. corresponding to y-2ay +x =a? by eliminating a. Sol. The given equation is y -2ay +x? =a? () Differentiating (1) w.r-t. x 2yy, -2ay, + 2x =0 wy te = a= Ma Substituting this value of a in (1), we get v0 (82) (m4) nm or Py? = 2yy,0y, +204 2y2 2992 4 Boy, +? or = yy? — Day, + 2 P= Dayy, +22 or (3? - 2y%y,? — deny, — BY gy Wee or a-a(2) dry 2-20 which is the reqd. differential equation. Example 37. Eliminate the arbitrary constants a and b from the equation y = ax + br?. Sol. The given equation is y = ax + bx? AD Differentiating (1) w.r.t. x Bas 2e (2) . _ dty Differentiating again = 7-7 = 2b (3) & ie From (3), be Ble g DIFFERENTIAL EQUATIONS AND THEIR FORMATION 19 dy dy ast ae t=O which is the reqd. eliminant. Example 38. Eliminate m from J1- x? + Ji-y* =m(-y). Sol. The given equation is {1—x? + f1-y? =mix-y) oo) Putting x = sin 6, =sin¢ (2) becomes cos 6 + cos @ = m (sin @~sin 6) or 2 c08 2*® cog 99 ~ im, 2 605 oF 8 sin oP 2 2 = # or == =2 cot? m or sin'tx—sin-ty = 2cot'm Differentiating w.r.t. x = which is the reqd. eliminant. Geometrical meaning of the differential equation a = f(a, y) of the first order and first degree. The given differential equation is 2 =f) -D) Let (x, y,) be any point in the xy-plane. Putting the co-ordinates of this point in (1), we get the value of # at (ep, Yys SAY My which is the slope (or direction) of the tangent at (x, y,). Suppose the point moves from (x,,y,) in the direction m, [i-e., along the tangent at (x, y,)] for an infinitesimal distance, to a paint (x), ¥.)- 20 GOLDEN DIFFERENTIAL EQUATIONS Let m, be the slope of the tangent at (x, y,) determined from (1). Let the point move in the direction m, for an infinitesimal distance to a point (,, 74). Let m, be the slope of the tangent at (x,, y,) determined from (1). Let the point move in the direction m, for an infinitesimal distance to a point (x,, y,). Proceeding like this, the point will describe a curve, the co-ordinates of every point of which and the direction of the tangent there at, will satisfy the differential equation (1). Since (x,, y,) is any point in the xy-plane, through every point in the plane, there will pass a particular curve, for every point af which x, 9, © will satisfy (1). ‘The equation of each curve is a particular solution of (1). The equation of the system of (such curves is the general solution of (1). All the curves represented by the general solution, ‘taken together, form the locus of the differential equation (1). 2 Solution of Differential Equations of the First Order and First Degree Definition. A differential equation of first order and first degree is an equation of the form dy Sea Aay) or Max + Ndy = 0 where M, N are functions of x and y. ‘TYPE I. VARIABLES SEPARABLE If in an equation, it is possible to get all the functions of x and dx to one side, and all the functions of y and dy to the other, the variables are said to be separable. Rule to solve an equation in which the variables are separable . od Consider the equation 2 = XY, where X is a function of x only and Y is a function of y only. 1. Given differential equation is x = XY. 2. 2 = X dx ie., variables have been separated. 3. Integrating both sides, [ e = J Xde+c, where c is an arbitrary constant, is the required solution, Note 1. Never forget to add an arbitrary constant on one side (only). A solution with- out this constant is wrong, for it is not the general solution. Note 2. The nature of the arbitrary constant depends upon the nature of the problem. Note 3. A constant is, after all, a constant, in whatever form it may be taken. Note 4, The solution of a differential equation must be put in a form as simple as possible. Note 5. Remember : (i) log x + log y = log xy (éi) log x — log y = log x (From Logarithms) (iti) n log x = log x* fiv) tant x + tan! y = tant [From Trigonometry] (o) tant x= tan“ y = tant 2 I+ay 22 GOLDEN DIFFERENTIAL EQUATIONS Example I. Solve the following differential equations : » ay . dy 8D _ get 4 tet SY 8 ett 4 etleer-y (iy set + xk i greet Gii)y—2 # =a Sol. (i) or & sere +x) | Note Separating the variables, e?dy =(e¥ + x2) de Integrating both sides, ferdy =[let+x9dz+e 3 or — e+ or e+e + 7) += 0, which is the required solution. a Gi Bn eea g gtr acts geile, oo sory + et sey Per le es = fx) Now proceed as in part (i). by i Separating the variables, 1 dx 1 ax ay atta ® 2. dy ii) you Gs avtta Fr ory-ayte (ata). 0 ae yray? atx yay) asx or [2+325| dy y* Tmay Integrating both sides, | Partial fractions) 1 or tog +0 (-4) tog cay) = og (a+ 2) +6 or logy —log (1 -ay) = log (a +2) + loge | takinge, = loge [See Note, ». or Jog 3255 = loge (a +2) log m tog n= og og m +g n=log mn | | a ttern or y= cla + x\(1—ay), which is the required solution. SOLUTION OF DIFFERENTIAL EQUATIONS OF THE FIRST ORDER AND FIRST DEGREE Example 2. Solve the following differential equations : (Ose stay de + arc yeandy-0 (2 log x +1) es wi) & = = ain y+ yoosy wa (iv) 3e* tan y dx + (1 -e*) sec y dy = 0. Sol. (i) sec? y tan x dy =—sec? x tan y dx Separating the variables, dy yor B any tany tan x Integrating both sides, 23 sec? tany or log tan y =~ log tan x +c, J Pace tog se) f(x) or Jog tan x + log tan y = log ¢ | taking c, = loge or log tan x tan_y = log ¢ Is log m + log n = log mn or tanxtany=e is the required solution. a dy __xQlogx+D ii) 2 = dx siny+ yoosx Separating the variables, (sin y +y cos y) dy = x12 log x + Dde Integrating both sides, J (sin y +y cosy) dy = J (2x logx +x) de+c=2/xlogrdx+Jxdxec or -cosy+ysiny—J1.ain y dy =2[ee. 5 - or cory +ysiny omy 02 [2 or ysin y = 2? logx +c is the required solution. d iii) [x-= B)eny dy o or woes or Poe Separating the variables, 24. GOLDEN DIFFERENTIAL EQUATIONS Integrating both sides, [¥-IG-s)e+e 1 or log y = log x + = + ¢ which is the required solution, (iv) (1 ~e*) see? y dy =— 3e* tan y de or (ef — 1) sec? y dy = Bet tan y dx 2 a Separating the variables 22°. fy = 2" ate tany Integrating both sides, sec” e ea ay =3f aes, or log tan y = 3 log (e* - 1) +c, or Jog tan y = log (e* - 1)3 + log ¢ I taking c, = log c or log tan y = log c (e* - 1° or tan y = c(e*- 13 which is the required solution. Example 3. Solve the following differential equations : (i) (2 — yx) dy + (2 + xy) de = 0 Gi) (e—y2w) de — (y — x*y) dy = 0. Sol. (i) (x? - yx?) dy + (y? + ay”) dx =0 or x1 y) dy + y? (14 widx =0 Separating the variables, 1-¥ gyi 4 deo y ¥ or 4+tlac=0 a Integrating both sides, 1 1.1 J 1) ays f(4+2) ae §Ge-F]o-Saed) 1 1 pr -—-logy-=+logx=c ¥ ¥ 1.1). or I (2 +) = cis the reqd. solution, Gi) (x—y*x) de —(y— x2y) dy = 0 or y (1-x)? dy ex(l-y) de Separating the variables, wy dy = arr dx SOLUTION OF DIFFERENTIAL EQUATIONS OF THE FIRST ORDER AND FIRST DEGREE. 28 Integrating both sides, J => yt aye le dete, ox 1¢ -2y or -g) arena 2) 3S sp dete, or ny egit-ahe- brag or Jog (1-9) = log (1-22) — or Jog (1 — 9?) = log (1-23) + loge | taking - 2c, = loge or l-y? sc(l=-x). Example 4. Solve the following differential equations : OMewereysl (if) x +") dx +91 +) dy =0 (iy B a eto ate (io) 4) 4 1207, sok geameneye tiene (e+ Idx ‘Separating the variables —— rae a 2 Integrating both sides, log (y + 1) = = + +c is the reqd. solution. (ii) x1 +y*) dx +91 + dy =0 dx=0 Separating the variables, Th Integrating both sides, f eee dy+ +7 Hay or F tog + y+ 3 1 hog (1 +22) = or Tag chay)a tog tat or Jog (1 + 92)(1 + £2) = loge a (+7 +450 is the required solution. (iii) The given equation can be written as Baler gat) em = didy w (eh 4 Avda or fet ae Integrating = Sa [+S] 40 or Qe = Se + Bx?+C where C= 6c 26 GOLDEN DIFFERENTIAL EQUATIONS (iv) The given equation can be written as eon Berernt dy _ de _ de = wae OF Oe OE Integrating = log (2-e%) = log (x + D+ => Jog (x + 1) + log (2 - 2” => log (x + 1) (2 -e%, = (r+ DQ-e)se* => (+ 1(2-e&) =C where C=e*. Example 5, Solve the following differential equations : oe .ee (ii) (e? + I) cos x dx + &% sin xdy = 0 (iid) x c08? y dx = y eos? x dy iv) sec? x tan y dy + sec? y tan x dx = 0 (vy & . tan y = sin (x + y) + sin (x-y). dy z+) Sol.) SY = 5a Separating the variables, ytd Integrating both sides, isd) aeft or y+ log y=x4logx +e or y—x = log ~ 4c is the required solution. y (ii) (e” + 1) cos x dx +e sinx dy =0 ° eat Integrating both sides, log (e” + 1) + log sin x = log c or (e’ +1) sin x = cis the required solution. x cos? y dx = y cos? x dy " Separating the variables, dy+ = dreo sine x cos? y cos? x Integrating both sides | y sec? y dy =x sec x de +e Integrating by parts, we get ytany ~Jtan ydy =x tana -Jtanxdx +e jor y tan y + log cosy =x tan x + logcosx+c lis the required solution. Separating the variables = dy= de SOLUTION OF DIFFERENTIAL EQUATIONS OF THE FIRST ORDER AND FIRST DEGREE 27 (iv) sec? x tan y dy + sec’ y tanx dx =0 Separating the variables 180 ty BEF gy 20 sec? y "> gee? x sin y cos y dy + sin x cosx dx = 0 sin 2y dy + sin 2x de = 0 Integrating both sides, - } cos 2y- } cos 2r=- fe or cos 2r + cos 2y = ¢ is the required solution. or or w & tan y 2sinGe-+y)+sin @-y) =2sin x cosy sinC +sin D=20in OP egg C—D Separating the variables, sec y tan y dy = 2 sin x dx Integrating both sides sec y = —2 cos. x +c is the required solution. Example 6. Solve the following differential equations : dy _1ty" Tee (ody fivx? dx +x fivy? dy=0 (Bangalore, 1996) ots iny-2 & ws(ne (v) (1 + aly dx + (1 + ye dy Sol. (i) See Integrating both sides, sin y + sin“ x = sin“ ¢ or sin? [x JI-y? +y JI-2? J =sinte or x fi-y? +y {1—x* =c which is the reqd. solution. l+y? dy _ dx l+x® l+y? Lex? Integrating both sides, tan“! y = tan"! x +c, or tant y-tan tz =c, ie stante | taking cy = tanec or or or tan? yox Lay which is the reqd. solution. or sc or y-x=c(l+xy) fi dy 2 gy tii) y-x B wares or y-3=Gr +n 7 * GOLDEN DIFFERENTIAL EQUATIONS in. a a (1 3 . . al ya seed Ses ee | * [Partial fractions] Integrating both sides, log (y= 38) = log x -3. } log (ar + 1) +c, hog 5 +loge I taking ¢, = loge Load co or ogy -B)=log gory 9 or y - 38+ Dace which is the reqd. solution, (ey y¥iee? oe dy=0 or wer dee ° Integrating both sides (0) pF of te v() Set aD ophp an) qhe -sta 1 3 1 =J 1 . pitfaw *Bydy where d= J te is ath, (sy e-sinktt4 ing? Jena 2 =—log t+ 1+? )+ fit y? =-le(}+ oe) in? iF tag Eo SF fat Similarly f 225 de=—tog VEE wee fit +» From (1), fis? + fie - log Levies? + fae SOLUTION OF DIFFERENTIAL EQUATIONS OF THE FIRST ORDER AND FIRST DEGREE 29 oF or frcat + Jeo? a tog LANIF#" 4 jog Levey” z ay Le y14x7) (14 V1+ 97 or few + fey? log Ee which is the reqd. solution. w (4 nyde+(L+yedy =0 or LY gy 414 dv <0 y * or (J+ 1}a+(F+1}ac-0 y x Integrating both sides logy +y +logx+x=0 or xey+logay=c which is the reqd. solution. Example 7, Solve the following differential equations (= 21 ~y) de = xy + yidy (ii) alady + 2ydx) = xydy ds (iy Bast ast te) tog (2 sax + by (Magadh 1998) (vhyde + (1 +2) tan“ x dy = 0. Sol. (i) (1-231 - y) dy say (1 +y) dy 2 or ty) gy doa I-y x or Integrating both sides 2 ~~ 27 Blog C1 —y) = log e— ote or —2y = logx +2 log(1-y) +e which is the reqd. solution. Gi) a(rdy + 2ydx) = xydy or 2aydx = xy - a) dy dys tae or y x Integrating both sides y-alogy = 2a loge +e which is the reqd, solution, B asta ves or or 30 (GOLDEN DIFFERENTIAL EQUATIONS or > rietahead or ds = xdx | Note carefully or Integrating both sides, J hog de® 2 3 log (1 = = 3 Fey or log (1%) =x3 + Se, = log e*” + log e = log ce™” or lee*ece™ which is the reqd. solution. to) ng (2) =a + => Bann [es log, m=x=m ea! Also, when the base is not mentioned, it is understood to be e] or Bee e or etndynettde Integrating both sides, ew ee aorta or bette =—abe, 9 or be tae =c which is the reqd. solution. ) yde + (14 22) tan xdy 20 dy dx r y Gee )tants 1 z or Hyde geo y tans Integrating both sides, og y + log (tan- 2) = loge or log (y tan"? x) = loge or ytantzcs which is the reqd. solution. Example 8. Solve the following differential equations : @U+dy+x fi-¥ de=0 Ge f1-y? de+2 dy=0 sai 7 : dy yleytd Gi) + yal tin By HAH =0 ) ma ty Zao. SOLUTION OF DIFFERENTIAL EQUATIONS OF THE FIRST ORDER AND FIRST DEGREE 31 Sol. (i) (+edy+eJ1-y? de=0 dy “ iy ii 2 de=0 +x? dy 1 2 +x? ty 27 Integrating both sides, sinty + Flog (1424 =0 which is the reqd. solution. ai) ei ye d+ > dy=0 or 2 dy +0" de =0 vi-y Integrating both sides, y J xorar] ware or -F ] dv Copy vee J etdene 2yu2 or +a ror or (x—Det= J1-y? +e which is the reqd. solution. (iii) x a +y=l or eBer-y dy ds or -y == Integrating both sides, ~log (1-y) =- 2 -e 1 log(1-y= F +e which is the reqd. solution. 2 wy ytytl 4 3) get ey 32 GOLDEN DIFFERENTIAL EQUATIONS or _# ,_# _. yrytl axel Integrating both sides, dy de |oSncleSa % de or f—.-+J-—“*- ety 48° [eed] 8 29) *4 2) *4 o a 2 tant 72, 2 tay 2 tan + tan’ Cy ° pa ae 2 2 or tan? B12 tag BEE Bo, a tarte {replacing ¢, by tan“ c] axed tent 5B tamete | tana + tan! ba tant 24 or Qye Qe4i ° 1-ab v3 or 2x4 2y+2 3 V3 3-@r+DQy+D or 2/3 (x+y +1) =e (2 -2r = 2y = dy) or (e4y 412 Cex -y- 21) which is the reqd. solution. (v) The given equation can be written as dy VL + 22X14 y?) + ey 204 9) bays Fi or 2 dy+2* dr=o jive? Integrating both sides, dy+{ 2% dene j ie 2 +J = +y ox Tay? — log LEVER” wes ee [For integration, see example 6 (iv)] fax or jive +fi+y® alo HEE +e which is the reqd. solution. SOLUTION OF DIFFERENTIAL EQUATIONS OF THE FIRST ORDER AND FIRST DEGREE 33 Example 9. won =e? and it is given that for x = 1, y = 1 ; find y when x =- 1. (ii) Find the equation of the curve represented by (y-yx) dx + (x + xy) dy =O and passing through the point (1, 1). ® ere Sol. (i) ter ee Separating the variables, Integrating both sides, mo) when rslysl = -else+e 1 lte? or cw= > =es-— e e From (i), Now when x =— or Gi 1-2) dx + x1 + dy =0 Separating the variables, 2+% dy +42 de =0 7 z or (E+1)a+(2-1)ae-0 y x Integrating both sides, log y + y + logx - x= Ai) If it passes through (1,1) c= 0 Iv log1 20 2sec? pt 2td tans SOLUTION OF DIFFERENTIAL EQUATIONS OF THE FIRST ORDER AND FIRST DEGREE 39 or =f qt +0 i+2e 4 2% 42242 stan stan = where ¢ = tan 5 andz tan > i da stan? or p log (1 +20) = f wap Peremtes Dee i x {tan = or 3 oe (24 2tan 2) =tan (und+1}+e which is the required solution. (v) The given equation is xde + ydy cay yds 0) Changing to polar co-ordinates by putting x=rcs@ and ye=rsing so that rs \etey? stant * dr Also an agonerene BaF cnosrensa ‘Now equation (1) can be written as ae yy “4H +90 67) a dr reas 0( $6 dr . 10s 9{ $f sin 0 rcos)- raino( 4 dr ; 1 Gp(cosO+sin® 8) (Fe 7? (cos” 6 + sin? 6) a or a or Integrating both sides, sin- £ =O4e cort-rins) sind sino rom] FF eas 8— rain) a8 40 (GOLDEN DIFFERENTIAL EQUATIONS r=asin(@+e) = (ae stant where reyeey? and O=tant 2. Equations reducible to the form in which variables can be separated. Equations of the form 2 = f (ax + by + c) can be reduced to the form in which the variables are separable. Put ax + by + ¢ =z, we have dy de #.1(E- ) athe ae or eb \dx i i 1d _,)_ <. Given equation becomes 2 ( z a) fiz) or Sa = ope) or & a softs Sey ing the variabl —#_ oa parating the variables [575 = which can now be integrated. Example 1. Solve the following differential equations : Gi} fe +P # =a (Indore, 1998 ; Meerut, 1997) Ge 2B age (Delhi, 1999) (ii) & ata vy + 32 ds (iv) 2 = tery? Sob (ery)? & wa? dy a dy dz Put xeys2, then lege & oe dz Given equation becomes z? (£ -1 dz a or G12 oo 2 Separating the variables, —-* — dz = dx Peta’ @ or [a de=dx Integrating both sides, 2-02. 1 tan! Z ax4e @ @ SOLUTION OF DIFFERENTIAL EQUATIONS OF THE FIRST ORDER AND FIRST DEGREE 4. + or xeyna tent 272 cate or yeaten SEY seis the required solution. . dy _ dz dy d& (i) Putx-y=2,then1-9 =F oor Ga Se =. Given equation becomes z* (-4) =a? & dx & a or a Separating the variables, 2 or z-e zta =xree Integrating both sides, z +4? 5 log or or -. Given equation becomes dz atte Separating the variables, = 7 1 Integrating both sides 5 tan! 5 =x +0 1 dx+y4+1 or 5 tant 3 7 ¥ + eis the required solution. , we (w)Putx+y=2, then d+ oo = Given equation becomes dz Bitet o Gatet Separating the variables, Sore Fe Integrating both sides, tan? z=x4c 42 GOLDEN DIFFERENTIAL EQUATIONS or tant(ix+yexte or x+y =tan(x4+c) is the required solution. Example 2. Solve ((c+y +) Zot Gi) @+y 41? Ber (Meerut, 1996) Bol. (Puta +y +12, then 1+ Y= ) =a +. Given equation becomes, z (¢ - or saad oo (-y)e-e Integrating both sides, 2 ~log(z + 1) =x +0, or a+ytl—logw+y414+ aztec, or y logs + y+ 2)=e,-1 or y ~logtx + y +2) = cis the required solution. Gi) Proceeding as in part (i), we have a(Z-)-1 or eHean tel #41 Integrating, we have 2-tan?z=x+e x+y+latanT@e+y+lexec ystant(rty+1)+C. where C=c-1. i) aot (2) azey = Sesine+y Putz+y=z, thon + 2a ” Given equation becomes $% —1 = sinz 2 1 or Goaedk oo (1-ghy) ena aa dz - or ae tht sing. Separating the variables, —“2— = de 1+sinz Integrating both sides, [ ws azee SOLUTION OF DIFFERENTIAL EQUATIONS OF THE FIRST ORDER AND FIRST DEGREE 43 or or or or or l=sinz Po dz=x+c cos? 2 or J (sec? z - sec z tanz)dze=x+0 tanz-secz=re#c tan (x+y) —sec(x +9) = x +c is the reqd. solution. Example 8, Solve the following : OB a sintery re0sle+y) ; - ay ae Sol. (i) Putx+y =z, then 1+ 9° =F . Given equation becomes, dg dey . G.~insinzt+eosz = or Ge = tease + sing Separating the variables, ——H L+cosz+sinz ~ az # z el 2 cos’ gt sin; cs, sec? 5 de : edx | Dividing the um. and denom. by cos* = (1+ tan } 1 1 2. Given equation becomes = 2 dz 4 ae Separating the variables, sidendx ot (hanes zed z a4 GOLDEN DIFFERENTIAL EQUATIONS Integrating both sides, z— 4 log (2 + 4) = 2 +, or + 2y-8—4 log @ + 29-34 Aaxtey or Qy — 4 log + 2y + Dae, 43 or By — 4 og (x + y + 1) = 2c or y —2 log (e+ 2y + 1) = cis the required solution. Example 4. Solve the following equations : dj (iy = cos e+») (ii) c0s Ce + y) dy = dx iy ay 1 _ dy xtytt Ge -y*? () de Ety (0) Ge + 2y)lds ~ dy) = dx + dy. ; dy_de Sol. () Putx+y=2z, then 1+ 7 a dy de or dx 7 dr 1 Given equation becomes ey % ~ 1 s0os2 & z or & 214 0082 =2 cos? z 1 z or cet 5 ends Integrating both sides, 2 tan — Femqeeste or tan SPP ace 2 is the required solution. (di) The given equation is ds dy TB E+” Put xt+ysz Differentiating w.rt. y de ae, ard ay .. Given equation becomes & -1 cas or Bw tecoz=200s§ or 5 sect = dee dy SOLUTION OF DIFFERENTIAL EQUATIONS OF THE FIRST ORDER AND FIRST DEGREE 45 Integrating both sides, 1 tan = 1B ee 271 2 or tan £5 « y+ cis the required solution. iia’ ay dy _,_& (ii) Putx-ys2, theni- i= Ge oor She l- oe <. Given equation becomes 1 1-£ e341 or —zdz=dx de Integrating both sides, 2 ~Feree, or Qeale-yP42ey=0 or Wet le-yP ae is the required solution. (iv) Put xeyez dy de dy _ de then 14 oe de or dx ao <. Given equation becomes aw, 2t1 dz _1+22 de z dx” 2 z 1 1 - or Tapert oo 3 (1-qdg) arma Integrating both sides, 1 gph (1 + 20-240) zi1 or Gg leds ae exse, or Qe —log (1 + 22) = 4 + de or Ba + y)— log (1 + 2x + 2y) dx or 2y= x) =log (2x + 2y+ +c is the required solution. tv) (x + 2yXdx — dy) = dx + dy or (x + 2y=1) dr = (e+ 2y + Ddy dy _x+2y-1 or "ipl sal) dy_de Putx+2y=2z, then 1 +2 5° = by 46 GOLDEN DIFFERENTIAL EQUATIONS or or or or x + Bc, or Gly —x) +4 log (Br + Gy - 1) =9e, or 3(y — x) +2 log (8x + By — 1 which is the required solution. Example 5. Solve the following differential equations : . dy 7 .. (x+¥-@ @ ae tise wo (74328) Sol. (i) The given equation is dy oe ow) dz Putx+y=z sothat 1+ # “= «. Equation (1) becomes foe or etdesdx Integrating — et =x +¢ or -let+ce or l+@toere0 dy de Gi) Putx+y=2 sothat 1+ oo = dy de or dee +. The given equation becomes (23 (&-1)-# z-b Jide J" 246 or dz _@+aXe-b)_ 2? +a2~-be-ab dz” (@+bhz-a) 2? -az+bz-ab or dz ___2(2?~ab) de 74 (b-a)e-ab or z += a)z— aiz-ab y eds 2 -ab SOLUTION OF DIFFERENTIAL EQUATIONS OF THE FIRST ORDER AND FIRST DEGREE a7 or [4 ® =a Integrating both sides, b-a 52a 2405 J pty aeaarse or 24} (—a) log? —ab)=2x +0 or r+y+} (b-a) logilx +)? -ab) =2r+0 or 3 Oa) log (er + y)?-ad)=x-y te which is the required solution, Example 6. Solve the following differential equations : xt B 4 xy 4 cosee (xy) =0 a 2. MAY IN-T Lg, dx x+y? Sol. (i) The given equation can be written as 2 (2B +y)+ emeetn)=0 ing sy = wy yu Putting ay =t,we get x So +y= 5 The given equation becomes de _ . dx ah ig Teosec t= 0 or sinede + =z =0 Integrating, we have ~ eos t+ = e or costuy)s sty =C where € =—e (ii) The given equation is y dy Betyg, xde* xt4y?a1 7 Putting af4y2=4, weget ova ay Ba x dy 1 dt y dy_ 1 ae or ede 2s'ds ™ g'dy Bede + <. The given equation becomes a. ae" 48 GOLDEN DIFFERENTIAL EQUATIONS Integrating, we have t+3 log(t-2Q4x2=c or xy? 4 Slog (x74 y?- B40 or 2x? + y? + 3 log (x? +? - 2) =c. TYPE IL HOMOGENEOUS EQUATIONS: Homogeneous Function. Definition. A function is said to be homogeneous of the nth dogree in x andy if it can be put in the form x =(3). Homogeneous Equation. Definition. A differential equation in x and y is said to be Y hey Al. homogeneous if it can be put in the form dx fal, y) , where f(x, y) and f,(«, 9) are homogene- ous functions of the same degree in x,y. Art. To solve the equation fo. fee » where f,(x, y), f,(x, y) are homogeneous functions of the same degree in x and y. Oreneantin'e Efe “+ file, »), file, y) are homogeneous functions of the same degree in x and y. Let fy) =x a, (2) iff = 2", (2) nf) aM) 4, w. a dx ve(2) (2) Paty won then % woe. .From@, & vo(D) o(2) ci (2) (say) Ai) z .. From (ii), ver Separating the variables, fw=0 which can be easily integrated. In the solution, putting v = ~, we get the required solution, Method :1. Puty=ur,thon 2 avse, 2. Put the above values ofy and in the given equation. 3. Separate the variables and integrate. 4. Replace v by > to get the required solution. 49 SOLUTION OF DIFFERENTIAL EQUATIONS OF THE FIRST ORDER AND FIRST DEGREE Gi) (2 + y9) dx-2axy dy =0 (Kerala, 2001) Example 1. Solve the following : (i) (9? yh dx + Oxy dy=0 Git) G2 + y°) dx + Bxy dy = 0 Ci 2 +99) H aay, Sol. (i) Given equation is (x2 — y2) de + Dry dy =0 day dy =~ (2-97) de xc) voy or ¢ or 2 Puty = ve, From (i), or Separating the variables, —2°= dp =-+ dx leu * Integrating both sides, log (1 + v®) = —log x + loge or = (1+? His log (1 + 09) =1og £ or 2 » 1+ = mo © z or xt +y? = cr Ai) or which is the required solution. (ii Given equation is 7° = = ® oat Putysux, then Go sutx Ge dv xt4u%x? 140? + From@, vex gs = We Ate? lee’ de QP Separating the variables, 2v dx dv=— 1-0? * ~ or - 1-v? Integrating both sides, — log (1 - v®) = log x + loge or log( — v2y"1 = log ex 2 (=v ytser 1 1 part oe zex 1 or GOLDEN DIFFERENTIAL EQUATIONS 50 or or x=cfx?—y?) is the required solution. 2452 (iii) Given equation is # = in Ai) Puty =r, then Pevaee . dy xteuts? +0? From), vr. Ge aut Lev + du? 1 Bo? ae De Bw Qv Separating the variables, 2v 1 6u Trae or 3 Trae Integrating both sides, 4 log(1 + 90%) = log x +c, or log (1 + 30%) = ~ 3 log x + 30, By? . or dog | 1+ 22) = 1og x3 + loge | taking Sc; =log ¢ x 2 4 gy? 24 gy? or Jog 29% wtoger? or SHB. ¥ x or x(x? + Sy") = ¢ is the required solution. (iv) Given equation is eal) Put y=ex then dy vx? v From (), ore Fade ie v-v- ve ite 1+u? Separating the variables, l+v" dv = oe . ot Integrating both sides, +> +logu=—logx+c or ~ ty + loge =— logs +e Bu? og x? x or - 2 2H logs +e SOLUTION OF DIFFERENTIAL EQUATIONS OF THE FIRST ORDER AND FIRST DEGREE st 2 or z + logy -log x =—logx +e ay or = Gp tory =e inthe required solution Example 2. Solve the following : Welhi, 1997) (ii) "ydx — (x? + y7idy = 0 it dy iy Ww) y+? =0G, 2 Sol, (i) Given equation is x & ay 2 * Puty =u, then a i) yt? From (i), ver ee or Separating the variables, * = # Integrating both sides, -ts-lgxte or v x == +logree y rhe is the required solution. s (di) Given equation ie 2% = 7 dx x+y? Put y = vz, then Boose tt , du vx? From), v+x 2 =a 2 1+v% dye v4 Sa ee ee Separating the variables, 2 dx At? eS vi z Integrating both side 1 ge thogvanloeree 32 GOLDEN DIFFERENTIAL EQUATIONS 3 = ca or ~ gt tlhe Fe logs te 2 or - 37 +logy—logx =-logx+e * or a + log y = is the required solution, (iii) Given equation is or wel) Put y= vx, then [Fat 2. From@), oer eB ye or Integrating both sides, sinh“ v = log x + loge or log (u + J14 v0?) = log cx o vt fit? = or pty ce or y+ fea y? =ex? is the required solution. Another Form of the Above Question Solve ady -yde = Jz +9? dz (Delhi, 1999) . ae 4 aw 9 . (iv) Given equation is P= cy oe pF wd Puty=or, then Daves % 242 ” Prom (i), v+ 222 = * SOLUTION OF DIFFERENTIAL EQUATIONS OF THE FIRST ORDER AND FIRST DEGREE 33 Integrating both sides, v— log u = logy +e y y or te | slogz te or — (log y — log x) = logx +c x x % elog y + ¢ is the required salation, Example 3. Solve the following : (ix? dy + y (x + ylde = 0 dy dz a wx Bay-z008 (2). or =~ +tan2 x x Sol. (i) Given equations Gr =— 7% wi) dy dv Puty sex, then det ay . dv ux(x+ux) «: From (i), vtent- yf or 2 Pa weey Separating the variables, du___ de 1 dx = ery Oe or | Partial fractions Integrating both sides, }[log v— log (v + 2)] = —log x +e, v or log oy =~ Blogs + 22, v or log 525 = —log x? + loge ¥ £ ye 7 bea OS or xty = ¢ (2x +y) is the required solution. ii) Gi ionis 2 2% ssin2 i Gi) Given equation is 7 = + sin 2 wi) a ge Puty =x, then dz .. From @®), ven tt aussie or or GOLDEN DIFFERENTIAL EQUATIONS a, =H wine . : 1 Separating the variables cosec udu = z dx Integrating both “des, log tan © = log x + loge Rls “ g logtan 3 =logex or tan which is the required solution. or or or or (Gi) Please try yourself. [ans sin 2 = (iv) Given equation is x 2% ay dog y— log + 1) dx Y 2 (iogt g 2 (toe 24.1) i) dy dv Puty=vr, then ae Pt “From (i), vara =u (logy+ Dav logu+y 28 sage 1 y.# 1 2 wala logy x Integrating both sides, log (log v) = log x + log ¢ = log cx logvecr or log 2 = ox is the reqa. solution. Separating the variables, x (0) Given equation io x2 x y~ 2 cs? (2) DY = E — cog? (2) li) Puty=or,then Yevge 2 . dv ee Ly — cos? =~ cos? From (i), uv +x. v-cos*u or x cos? Separating the variables, oc? v du = - Integrating both sides, tan v = — log x + tan (2) +1og 2 is the required solution SOLUTION OF DIFFERENTIAL EQUATIONS OF THE FIRST ORDER AND FIRST DEGREE 55 Example 4. Solve the following : dy Sry ty? . dy . oF Wi (e+y) Ger (e-y=0 (Git) yds + (ry +29) dy = 0. Gi con is 2. 2+ ¥* . Sol, (i) Given equation is = SSP Ai) Puty =ux, then Bove e 7 dv _ Sux +u%x? _3u+0? + From(i),vtx, =P , dx 3x? 3 do _ 3v+u® v or Sey Separating the variables, 2% a Integrating both sides, — or - 2 Put y = vx, then From (i), vtr—- = = or Separating the variables, 5 Integrating both sides, i g loge +1) +tant v= —logr +e or belt ++ tant 2 = logx +c xtay? 1 2 = pit ee or 2 log + tan x logx+e 56 or on or or or or or or 1 1 ¥ i 2 yy%)—2 yo g bee +y*) g log x + tan x logx +c i 2 F tog (xt + 9?) + tan 1d = (ii) Given equation is ydx + (xy +22) dy = 0 Puty = vr, then From (i), dy a vtx a log (x? + x2) — 3 - Blog x + tant =-lgr+e GOLDEN DIFFERENTIAL EQUATIONS =c is the required solution. vu vel Integrating both sides, log v - } log (2v 4+ 1) = -logx +e, 2 log v — log (2v + D log v? — log (20 + 1) 2 a ‘ 6 24178 2 y fty c yf eae S s/s x 2logx +c, — log x? + loge or vie wet 2 or 2 £ xOyta) x xy? = clx + 2y) is the required solution. Example 5. Solve the following : dy _ very) or 2 (iii) (1 4°19) de + ot9 (- 2) dy =0. y Sol. (i) Given equation is Put y = ux, then From (i), vex wg dr @ O avers dx do wrt y) ot dv dx ax? vx(e+ px) _ w+) wf) | Partial fractions sali) SOLUTION OF DIFFERENTIAL EQUATIONS OF THE FIRST ORDER AND FIRST DEGREE 57 or dv use ae” 2 Separating the variables, or 2 ga or wep 7S Integrating both sides, 2flog (v - 1) —log v] = log x + loge or or or iis the required solution. (ii) Given equation is sali) Put y = ox, then «From (i), ut+x. or or sin-! 2 = log « +c is the required solution, (ity Given equation is (1+ 7) de +9 (1-2) dy =0 a) x Put Sav ie, xeuy | Not carefully dz do 7") ay Now from (i), (1 + e*/”) dx = er (- 2) dy y, 8 GOLDEN DIFFERENTIAL EQUATIONS ef =v) jor ' 1+e or y pr v+e" l+e’ ‘ 5 1+e* dy Separating the variables, “> du = - = ote y Integrating both sides, log (v + e*) = ~ log y + log c or logo +e) =log= or vent y y or = petra or rey ac y y ie the required solution. Example 6. Solve the following : i Yevsind int 2), % ; (i) { xcos % + y sin} y—-| y sin - xcos~] x =O (Lucknow, 1997 ; Kanpur, 1996) x x © x)” dx pr way dy x? + dy? Giye By ax fet 9? (iD) et ae? + =0. (iv) xin (2) a= [>an(2)-» dz to [ean 2— y ae? 2) de +x seet 2 dy=0. * = Sol. (i) Given equation is (sin(2}-» x. ay remy et Px wi) dx x sin sin x x SOLUTION OF DIFFERENTIAL EQUATIONS OF THE FIRST ORDER AND FIRST DEGREE 61 or or or or or de dz From (i, v+x #2 = Usinen} sin dy _vsinv-1_ "de sinv . . . dx Separating the variables - sin vdu= = Integrating both sides, cos v = log x +c cos x = log.x + ¢ is the required solution. (v) The given equation is (xtan 2 ~y ee? 2) des coe! 2 dys r ¥ x x see? 2 dy= (voce? 2-etan) ae * x = 29 x 2 2-xtan> tan yO ey w ds eee? 2 nec? z © dy de Put y =ux, then entre = Equation (i) becomes tanv sec? v => Integrating, we get log tanv + log x= Jog (x tan v) =e or xtanv =e rtan 2 =C where C =<. Example 7. Solve the following : ., dy *-2y @) 028 - 3x7 dx = (y" - dx°y)dy GD Get deny =? tity 2 fay -2) B y=, Sol. (i) Given equation is wi) Put y = vx, then 62 GOLDEN DIFFERENTIAL EQUATIONS From (i), var 2 epee = . ode ait dv _ 1-37 1-30? - vt +30? 2s -os or dx —30 . . dx Separating the variables, dv= xy . | vt_ay Integrating both sides, ~@ du=logx+c, 3u ly - 4 3 2v Now J jot eng uy = os) eee 1 “3 =o g log (=o) 9 1+ 1 381 -+ 3 2 qlea-v9-5. 7 =- 3 Liga v9) +B log + 1 4 =~ log (-v4)| 7 baglann( a-v7)* wn jog [are2* 4 log (+e)? 1-v* or or or Y or te we = c(x?— 92) which is the required solution. x-2y rn i) Given equation is det deny 7 wl) dy do i Puty=us,then Go =u +s. 7 SOLUTION OF DIFFERENTIAL EQUATIONS OF THE FIRST ORDER AND FIRST DEGREE 63 +. From @), vee. + or or Separating the variables, Integrating both sides, J dv + logx =e, or 2f oa neets |] cor i dv + logx =c, or 2, Bog 3*2 +2 tog (1%) + logs = or Jog 22242 og ca - -+logx=c, "or tog 422 « toga v4"? + logx ec, or or or d+)? inet fi-e ‘Squaring and cross-multiplying, (1+ v)'x* = ¢,? (1 -v) - (ot) eae or (x+y)? = elx — y) which is the required solution. sy «on ig GY y F ) Gi tion is — =0 os (iii) Given equation is de + 2-2 (id : dy do Put y= ux, then = v4 x 7 ox From (ute, 32+ = — 20 Wrr? 2 or or or or or or or or GOLDEN DIFFERENTIAL EQUATIONS v re to dv v+20/v one or “dole Separating the variables, aes dv iv = [} 1 Jeo vo QeJv Integrating both sides, f (2- geet) + logs =e log u- 1 + logs =c or loge + 7p + logs =e T 4 2 toe t+ |E +g eae or lngy-lnge fE sags=6 x y y x logy + E =c is the reqd, solution. Example 8. Solve the following : Wadz + ydy = m(xdy ~ yd) (ii) (22 + 2ey — 92) de + G7 + Qxy -22) dy = Give B oF -1. Gol. (i) Given equation is xdx + ydy = m(xdy - yds) (x + my) dx = (mx -y) dy dy _x+my de omx—y Putyses;then Beaver & . From (i), vee Stmus ime mx-vx om-v iy l+mv 2,2." dx m-v m-v Soparati arial parating the variables <~—> [ m1 de |e Teoh 2° Ise * Integrating both sides mitan" v. flog (1+?) =logx+e SOLUTION OF DIFFERENTIAL EQUATIONS OF THE FIRST ORDER AND FIRST DEGREE y o mines 2-2 (te 2) ate or mttan* 2-4 flog 2 + y2)~ log 24} =logx +6 or m tant £— 5 flog 2+ 92) — Bog x] log x +6 or man! 2-2 tog (xt + 92) = eis the reqa. solution. Gi) Given equation is o&, xt9-¥ +2ay-y ye + Day Paty =r, then & ays $2 dy #2 4 2ue* —v2x? + From()), uts 5 + haya og? dv or vex ta or Oe, Le dun vl pea Bu? =u dx vt 420-1 du vtutued or “ae * ear Separating the variables, 2 *22-1_ q,,% 29 4uel x 2 4 ay or i td-1 yg + DUD x vt +201 Integrating both sides, J oy GP ap dos | w+Qu-1 A Bu+C @+ Dey Fetl oF +1 Multiplying both sides by (v + 1c? + 1), v8 4 2y-1 = AQ? + 1)4 Bv+ O41) Puttingy=-1in(9), 1-2-1=AQ+1) 0 wAs-1 Equating co-effi. of v?, 1l=A4B -B=1-A Equating constant terms, -1=A+C -,C=-1-A=0 v+Q-1 0-1, v +1 wt 4 Let or or or or or or or GOLDEN DIFFERENTIAL EQUATIONS - 2u ~. From (2), = mn, (SH Py) ae toe ¢ = log (v + 1) + log (u? + 1) + logx =e, x(v?+D xv? +0, log ———— =k = Bed Be v+i ~° 2 «(2 ea}-e(20y) z * x+y? = elx +) is the reqd. solution. a Oi a dy _¥-y¥ (iii) Given equation ia S° = Puty = vx, then Hose + From (1), Separating the variables, 2v dx Troasan 2f1 4 ds sle-tle- = Integrating both sides, (I) | Partial fractions EL J log (1-20) —Iog (1 4 0)] = og x +c, ~ } log (1 - 2v)- § log (1 + v) = log x +c, Jog (1 — 2v) + 2 log (1 4 v) = 8 log x 3c, Jog (1 - 20) + log (1. + vy}? = —log x3 + log ¢ log (1 - 201 + v)? = log ¥ (1 ~ 20Y0 + v= x (8-3) Jo (= 2y kx +9)? = is the reqa. solution, Example 9. Solve the following : Gi) 2e%dy = be? + 92) de (iii) (7? — Byside + xy? — x9) dy =0. di) (2? + ayy = (2? + y2) de SOLUTION OF DIFFERENTIAL EQUATIONS OF THE FIRST ORDER AND FIRST DEGREE a ‘Sol. (i) Given equation is ali) Put y = vx, then . dv _ xt sets? 1 From(), vex == aD dy _leu® ) i+u*—w w= or 22s 3 5 . . dv__de Separating the variables, "iy = 5 Integrating both sides, ~ Arp e re, 1 or ~ sh = fee + xl ae or a sEgr plore, or EE elope eee or Qe = Wy — log 2 +6) which is the reqd. solution. . , dy ty . (i) Given equation is SY = Ty i) dy dv Puty = vx, then Fevers . dv xt sets ¢. From (i), ore ee dy lsu? or Oe Teo do ise ” ae" ite or te owe or (-1 BJ & To x Integrating both sides, ~ v ~2 log (1 v) = log x +¢ or ~ 2-2og(1- ) toga x or - 2-2 log =—* = log +e x x 68 GOLDEN DIFFERENTIAL EQUATIONS or ~ & ~2 log (& ~y) —log 2] = log x + € x or logx = Blog tr-y)+ 2 +6 which is the reqd. solution. a) Gi ionis 2 4 Yn . (i Given equation is 2 + ae i) = dy dv Puttingy eux, Pevex From (i), or or Separating the variables, 2 Integrating both sides, [ 45 dv + log x = cy 1¢ (207-1).20 . or 6) tw — du + logx =c, | Note this step 1p 2-1 or B! ape Wt eee ey whore t= 0? or aS (P+ Alerter a | Partial fractions or & [og ¢ + log (¢- 1)] + log x =e, or log tt - 1) +6 log.x = 6c, or log vu? = 1) + log x* = Ge, or log fou? 1). 28] = log ey or viv? 1) xe, 2/42 YU _4)s8- . uae or vot see, or y Gor =e which is the reqd. solution. SOLUTION OF DIFFERENTIAL EQUATIONS OF THE FIRST ORDER AND FIRST DEGREE Example 10. Solve the following equations : 2 wP+t-5 Gi) (2 499) dee y+.) dy 6x? +297 dy 2am) wi) = Fite 60) et a e)xtz—) & axe+y) Wi) ale -y) dy +y? de= 0. Sob. (i) Hint. Putting y = vr, the given equation becomes dy _ de wo- 2 ~ x Integrating fa 2] ¢o=top2 ee 2 log x + 2c" = 2 log x + loge 2eet oo = 1 7 y-2esay Gi) Hint. Putting y = ex, the given equation reduces to or Tey: Integrating J(-1+ ho nmmsee or =v =log(1=0) = logx +e x 2 I = + or log ( } logx+e or ~ 2 ~log (x9) + logs slog + 6. (iii) Hint. Putting y = vx, the given equation reduces to dv 6-v-2F er) 1+4v or Rave Oy 240-6 x Integrating log (207 + v - 6) = — log x + log 2 Yt _gek 24 xy - G2 = or 2at bE or By? + xy — Ox? = cx, (iv) Hint. Putting y = vx, the given equation reduces to dy, 24+30+0* 4 v?+1 de dee ve +02 70 GOLDEN DIFFERENTIAL EQUATIONS Bu? +3 —— dy +I = Paden M+ lexHe q et? ster e ogeed log (v3 + Bu + 2)+ 8 log x = Be’ = loge log x" (0 + 3u + 2) = loge 3 or #(4+2+2}<¢ or 4 ely 423m. Integrating + 888 (v) Please try yourself. [ne stop d=¢| x (vi) Please try yourself, (Ans. y = co] TYPE OIL EQUATIONS REDUCIBLE TO HOMOGENEOUS dy __axtby+e Art. To sob Bet byte fo Ive the equation 3 * Oxtbyte Case I. When seh Putz=X+h, y=Y+k ld) where X, Y are new variables and A, k are constants. ‘Then dr = dX, dy = =. Given equation dY aX +A)+ WKY +A te aX * aK thee +hee d¥ _(GX+bY)+(ah+bk+o) . or GX * @K+bY +@h+bk+e) i) Choose &, & sueh that ah+bkec=0 wha bane | Why ? To get rid of the constant terms. Solving by cro:. .wultiplication, wii) +. Eqn. (ii) becomes, ax aX4b¥ which is homogeneous in X and Y and can be solved by putting ¥ = vX. In the solution, putting X = x—h, ¥ = y —h [where h and & have the values given by (iii)], we get the reqd. solution. I, however, S =F ie ab! —a'b=0 4 and k both become infinite. Hence the method fails. SOLUTION OF DIFFERENTIAL EQUATIONS OF THE FIRST ORDER AND FIRST DEGREE 1M ‘Case I. When Let then ax +by+c max + mby +c’ or dy | _ax+byte a dz mlax+by)+c" “ dy dz then a+b. ae 1(dz Bete) From (i), i(¢- g-as __ Wate) +almz +e’) or = ? mz +e Separating the variables dx de Weted+alme+c’d which can be easily integrated. In the solution, putting z = ax + by, we get the required solution. Example 1. Solve the following = (Agra, 1996 ; Bangalore 1996) ci) @ .229tt MD Ge ey 2" as, «dy _ x*2y-3 Sol, () Given equation is 2 = 5 [sierea = 1.0"=2,8=2,8'= 1% 2 Put reXshyaYok a dx = dX, dy = d¥ d¥ X+h+2V+h-3 ax | UX+h)H¥4H)-3 dY — (X+2Y)4(h+ 2k-3) dX ~ QX+V)+@h+k-3) Choose A and k such that h+2h-3=0 and th+k-3=0 «From (i), or wwf) Solving by cross-multiplication, —— GOLDEN DIFFERENTIAL EQUATIONS => he=lke=l wn liii) a _X+2¥ dX 2X+¥ a dv Put ¥ = 0X, then mk i dy _X+2X _ 1420 + From (iv), 0 +X. = - (0 +X. SK Gee Eqn. (iii) becomes, iv) dv Xx Separating the variables, 3 av = & atv ak te do =f ote Integrating both sides, f 2 Bro or Teoasny = eeX+e 2-1 241 * Sledatcs*atecy |= meeea 1 or Slmtst or Flog (1 +0) — § log (1-v) = log X +e, or log (1 4 v) 3 log (1 — v) = 2 log X + 20, or log (1 + v) — log (1 — v= log X? + log c lev lee \ slog ext, th or 8 Gay 78 awe y les 2 or iat or Ce woe (-3) or X+Y=c(K- YP or z-1+y-Lecle-D-Q-DP or x+y-2=cx—y? which is the required solution. -" . , dy y-*41 (i) Given equation is = 555 afi) & [Horea=-o=3a'=15'=12 SOLUTION OF DIFFERENTIAL EQUATIONS OF THE FIRST ORDER AND FIRST DEGREE 3 Put xeXthy=Yrk then dx = dX, dy =d¥ From(y, oY 2 Yte- Me Wad CX+VHCALRTD (ii) ’ dX YskR+Xth-5 + Y+(h+ k-5) “ Choose h and & such that -hek+1=20 and A+k-5=0 whence h = 3, k = 2 (iii) .. Bquation (ii) becomes, Sy = =~ ae wlio) Pot =X, then Deux X+0X os From (iv), +X. 32 = X+oX ~ x dy itu =ltv-v-v? l+v* Oe Tee PT ee Y iy a . . lt Separating the variables, >——y du =— > Integrating both sides, f 14 ay -f Bee ve 1 or Jeera a z du =-logX +e or tant v + 4 log (1+ u*) == logX +e 4¥ i ¥? or tan! E+ Jiog{1+ 3p) =~ lore 4X 4) x+y? or tan? + 5 log Sy = log X + or tant L +2 hog ox? + ¥9~ log X= —log X + ¢ or tant vp 9 2 log X] =—logX +e or tant NaF log 2 +¥8 =e or tant 2 : Dt 3% +G-2lac ls Xex-hex-3 Yey-hey-2 or tant 22 4 2 tag 2 #9? Gr -4y + 18) = iwhich is the “quired solution. 4 GOLDEN DIFFERENTIAL EQUATIONS dy _#-y4l de xty~2 b [Herea=10'= 18 =~ Loran tes a bi Put xeX+hy=¥+k “ dx=dX,dy= A- R41 (X- (h-k . ty, TE SAMA esl Wehbe D (iii) Given equation is = dk ~ R+hy+Y+W-2 * Ka V+ (heh) Choose A and & such that h-k+1=0 and Aeh-220 whence h=dik . » a XxX aFromi), Fee X = we we Put Y = oX, then gx 7Utk X-0X 1-v «From(io), +X. Xeix"ice x, dee Jevev-v? 1 ap-e?@ ve D-1 ak Tsu” Tee Ito sl . , p41 aX Separating the variables, Fema 1 _Q~-2 aX or 2’ Pana’ X Integrating both sides, 4 log (v + 20-1) == logX +c, or log (0? + 2v - 1) =— 2 log X + 2c, =— Blog X + log c or log (o? + 20 — 1) =— log X? + log e= log 37 24 Qy-1= fp or tee et, or v4 Qe-1= ye or sat 1 = or Y?+2XY-X?=c or G- $F +20-fy- P-a-fsc or y= dye $4 2oy-gx-b yeh) -G x4 Dec or yr -3y+ $4 dy -3r-y+ fax -dee or yP4 dey —2t Oe dye f or y? + Dey — a? 2 - dys A where A is an arbitrary constant, is the required solution. onli) wii) wo (dii) a(iv) SOLUTION OF DIFFERENTIAL EQUATIONS OF THE FIRST ORDER AND FIRST DEGREE Example 2. Solve the following : @ty-y? (i) e-y) dy =(e4y + Dade wo ®t Se titiy 2x 4 8y =) % + (Be By-6)= 0. Sol. (i) Given equation is % 2 styel x-y , , a,b [Herea=40 =Lbehb'=-42 3% Put aaXthyeYrk then ds = dX, dy =d¥ » From (i ay M+h+Vsk+1] X+Y)+(hekeD « From(i) ox = Gam XY +h-B Choose A and k such that h+kel=0 and h-k=0 whence he-}kend . dy _X4+¥ v. (éi) becomes KRY a de PutY=oXthen Gy a0 +X Gy dv X+vX_lsv <. From (iii), vtX oy = Xoo 1 x fe Ate) eemetet _itet “dX "1-0 1-0 1-v Separating the variables, —" dv = & le x [Z 1 2] ae Tee 2 sel Integrating both sides, tan! » — 3 log (1 + v?) = log X +e aYi1 y? or tan? ¥—3 og [1 Fe) =X te or tant ¥ 2 og + ¥4)— log X4}= log X-+6 a¥l 2 or tan? ¥ - 2 (og cx? + Y2)~ 210g X1= log X + € Yiu ivi 24 2) = or tar 3 log (X? + Y*) = ¢ 75 wulé) Ci) ~liii) fs GOLDEN DIFFERENTIAL EQUATIONS or tant 24 Dog tah ey Wee o tan 22h A glins B+ DH ee 1 2y+1 or tant 21 liggataytersys dee which is the required solution. 2) Ge jonis 2% = ety-v* ) (id Given equation = 22— 2 a) Put eoX+hy=¥ak then dz = dK,dy=d¥ ot yp . ©, EL HAF Y tk? K+ rhe k-D i) aX Aen AKA 2F ‘Choose A and such that A+h-120 and h-2=0 when A=2,k=-1 2 «Gi becomes, a ae di) Put ¥ = oX, then FD evex, & . a dv _ (+x a+)? “From (i, v+x 2-20 Gee dv dtu?) Gta Xx a” 4 , . 4 ax Separating the variables, Ay dv = Integrating both sides, 4 J (1 - vy? du =logX +c (vy? or 4. Say ceX+e or pip cimexee or y = logX+e = Y aX M “t 2 on Reyes or TS = dogte-Ave Mx-2) © Gogg sp Thse-2+e or MED tog (x ~2)+ cin the required solution. { x-y-3 SOLUTION OF DIFFERENTIAL EQUATIONS OF THE FIRST ORDER AND FIRST DEGREE 71 a eas aon ag AY, Bat By-5 Gii) Given equation is 7 = — Seas [Herea=3.0'=36=26'=3 a tek a8 Puts=X+hy-¥+k — thendx=dX,dy=d¥ AY WK+N4 UY +h)-5 dX ~~ UX +A+HY+)-5 oe AY __ OX +204 Wh + Bk-5) i) dX (2X + BY) + (2h + 3k - 5) “ Choose A and k such that BA+2k-5=0 and 2%+3k-5=0 = A=lk=l. oui) =: From (i), +. (ii) becomes a7 aKa iif) Put Y= 0X, th Dex. & = oX, then ax 7 Oth ay +. From (iii), v+X.- do | 3420 _ 3+ 20420430" __ Su? +av+3 Xa aa 2430 3042 Separating the variables, P+? — dy = - FE but 2dX — pes * Baden ’* x 7° Integrating, log (Gu? + 4v + 3) + 2 log X = log ¢ or log (30? + 4v + 3) + log X? = loge or log (30? + dv + 8) X? = loge 2 or (v2 +404 90K2=¢ or (Sr +t s}x-e ek or BY2+4KY+SX%=c or -Siy— A) + dx hy) +B -APE =e or 3-1)? +4 - DY - 1) + 3-1)? ac or Sx? + dey + Sy? —10e- 10y <¢- 10= A. where A is a constant, is the required solution. Example 3. Soive the following : dy 2x~-y+I Oe? xt By-3 (ii) (12x -+ 2ly - Bde + (d7x + dOy + Tidy = 0. Sol. (i) Given equation is dy 2x-ytl an? ar iy-S ui) Herea =2,a'=1,b=-1,b'=2. Se r 3 GOLDEN DIFFERENTIAL EQUATIONS Put xeaX+h,y=Vak then dx = dk, dy = d¥ |. From (i) a | Kah) (+41 _ OX-V)+@h-k+D " aX” XK +h+AY+ WB K+2V+ (hs Bk- 5) Choose h and & such that di) Qh-k+1=0 and A+2k-3=0 - nedeel eB a dY 2X%-¥ on ii) becomes xe wAGii) Put Y= vX, then x iit ae «. From (iii), vtk iy X+2K 1+ 20 xf 2 _2-v-v") or ax * “Tsay or ett du = 2 x vtv-1 “x Integrating both sides, log (0? +» = 1) == 2 log X + loge or Jog (0? + v1) =~ log X? + loge = log 5 or Pro-le gy vy? o¥ c lor gery izge oof YeKY-Hec a RE + @ Aly -B)- AP ae (oF Bb B}- 8) IS ISS BJ 4. 49 747 2) br ¥ Bt gg gt gt gg tea re or pemetinctyee- or yary-2-x-3y nA where A is an arbitrary constant, is the reqd. solution. (ii) The given equation ia dy 12x421y-9 . de 7” Gxt Wyt7 i) b [Herea = 12,0 =47,b=21,0'=40-, Set 4] a’ 2B’ SOLUTION OF DIFFERENTIAL EQUATIONS OF THE FIRST ORDER AND FIRST DEGREE 9 Put xeX+hy=Vok then dx = dX, dy =d¥ a From(), 2 -- 2R+M+2UV + W-9 _ GAN+21Y)+0A+9R-9 . dK aX +h) +40Y +A)+ 7 ATK+40¥)+(4Th+ 40847) Choose A and & such that 12h+21k-9=0 and 4h+40k+7=0 => h=-lk=l +. (ii) becomes: wii) Put then a WX+2WK _ — 12+2W 1 From (ii), v+X 22 =~ Ga pagox 77 aT edo xt, sew _ _ 12+ 68v+40v? al ak * "474400 47 +400 A7+40v 4, a 12+ 680 +400" ©“ X or 400 +47 9 10v? + 17043 x 400+ 47 _ Adk * (2v+8MGu+ bX 2 15 4dX 5 , or (aa 25] dy =- x (Partial fractions) Integrating both sides, we get log (2v + 3) + 3 log (5v + 1) =~ 4 log X + loge = log (20 + 3X5u + 1)" = log Pa © 3 c = @+Gvra Se = (2Z+a](0Z+1) “a > (8X + 2¥) K+ 5YP ac = (3(e4+ 1) +2y-D (4+) +5y-DP ee = (Bx + 2y + Die + by 4 =e is the reqired solution. Example 4. Solve the following differential equations : (i) @x-y + Ddxt @y-x-Ddy=0 (ii) (8y~ 7-3) dx + (Ty -3x- 7) dy =0 ity &% =p AHH 20 dx” Ge 2y~10 80 GOLDEN DIFFERENTIAL EQUATIONS Sol. (i) The given equation is (Qe-y + Dde+ Qy-x-Ddy=0 dy _ 2x-y41 al Ge x-2ytl ~@) [Herea=26=-10'=40'=-2:. 2,2 a8 Put x=X+handy=Y+k so that: dx = dK and dy = d¥ . AY _ AK +A)-(V+A)41_ OK~ V4 Qh—-k+ D + from (LOK ~ Kah) AY+M41 K-2Ve(h- Dee D (2) Choose A and k such that Qh-k+1=0 and h-2k+1=0 = ha-},ke} a x Y + From, = ¥-ay (8) x dy Puty=0X sothat So =u +X 5 . dy _ 2X -0X + From (9), 0 +X Say xe Be koa? or ax 7 1-207" * =p or 2-1 ay ag & vt vs] x Integrating both sides, log (v? =v + 1) == 2 log X + log ¢ ¥ ¥? —x¥4x? or bos (Sp- +1) =e Sr or SF or GHP = (e— Ay h) +e hile oe O-PF-G+ Jy P+ Pee or Beaytylex-y+ dec or oaysytex-y+C=0, — whereC=}-c. Gi) The given equation is (By - Te - 3) de + (ly 8e- Tidy =0 or dy __ Tx-3y+3 0) dx" Be Ty 47 [Herea=7,b=-30°=3,6/=—7. S32] oF SOLUTION OF DIFFERENTIAL EQUATIONS OF THE FIRST ORDER AND FIRST DEGREE 81 Put xeX+h ‘and yaYok so that dx=dX and dy=dY a From(t, 2h. TK+M-BW+H+3 __ (TK- BY) +CTA+ BA +3) ° dX BK +hy- UY +k) +7 (8X - TY) + (8h - Tk +7) Choose A and & such that Th-3k+3=0 and 8h-7k+7=0 = h=Okel (2) 8) From (2), Put Y= 0X so that SY =v +x 2 .. From (3), vex == or or or Integrating 5 log (v + 1) + 2 log (v - 1) = - 7 log X + loge 5 2 or logo + Fw log sr or ($+3) (2-1) - 5 or WW +X8(Y-K ec or Y-k+e—-AB Iy-B)-@- Pac or (x+y- 18 r-y+ Pec. (iii) The given equation is dy 2r+9y-20 de” 6x+2y~10 oft) [Herea=2,4-9,0'=6, bi =2 S43] @ Put xeX+h and y=Y+k so that dx=dX and dy=d¥ 4. Prom(1), 2 = 2RtM+HY 4-20 _ OX +9¥) +(2h+9k—20) - * GX = GX+h)+2¥+8)— 10 ~ (6X + 2¥) +(6h + 2k - 10) Choose A and & such that 2h+9k-20=0 and 6h+2k-10=<0 wo) => =1k=2 d¥Y _2X+9Y + From (2), ay 8) Put Y=oX so that FT ax B® dX dX 82 or or ggg GOLDEN DIFFERENTIAL EQUATIONS dv _ 2X+90X From (3), veX ye @X + 20x xe dy _ 249 2+ 8y — Qu? ak 64m” HBr 340 1 dk 14 1 dx aaa 2X (eaeesh)e=s- x Integrating } log (1 + 2v)- log (2-v) = } log X +c log (1 + 2u)—2 log (2 - v) = log X + 2c’ = log X + log c, where log ¢ = 2c’ 1+ 20 1420 = log cX = 08 a yt = 108 eau? . 14 cex(2 ¥) or X42¥=cQX-¥P x (x - h) + Ay — k) = [2x -A)- - F x4 2-6 acl2e~y?. Case of Failure: 5-3, Example 1. Solve : : dy __etyed Gdn + by + B)de = (Be+ 99 +2)dy Gi) B= (Lucknow, 1998) (iii) x + By + dx — (Ax + Gy + 5) dy = 0. dy Axe6y+3 4 dy | 22a +89) 43 de Gxt 9y+2 de 3(2x+3y) +2 Put = 2e48y= wathon2s3 22% o a From, 3 a |= 25 Sol. (i) Given equation is Ai) dx 3242 dz 6z+9 fo. dx 3242 : : 8242 Separating the variables, i2e+13 15 2 12z + 18)32 +204 [E-S-ats]@ a 32438 Integrsing both sides, 5 6249 4 122413 o SHE ge dx "3242 a+ dz=dz z 8 ‘TD 1 jog (122 + 13) = +0, 122 — 5 log(12z + 13) = 48x + 48c, 12(2x + 3y) — 5 log (24x + 36y + 13) = 48x + 12(8y — 2x) — 5 log (24x + 36y + 13) = c is the required solution. SOLUTION OF DIFFERENTIAL EQUATIONS OF THE FIRST ORDER AND FIRST DEGREE 83 dy __xty+1 — (rt+y)41 (i) Given equation is JF = ed bend fi) bts eyentin te Hoey stmt, Bando HEH . : 224+3 Separating the variables, 3°" dz = dx 424TH or [Bed gty]enee met a Integrating both sides, 22+2. 2 log Qe +4) =3 + or Gz + log (32 + 4) = 9r + 9c, or Gls + y+ log (Br + By +4) = Or +e or S(Qy —x) + log (Bx + By #4) = is the required solution. 5a) Ci dy 2rt3y+d_ 2r43ye4 (iii) Given equation is > = Getess6 Ris tiyheb Put 2x + 3y =z, then wi) dy de 248 ae or 7 1fdz z+4 2. From @), 3[2-2 eS or deg B+ dz _12+22 dx 2245 dv 2245 Separating the variables, 227° ge ode \ —— Tz+22 | Tz4+22)2245(2 | 29 1 | e+ * i 9 aga] =e | > , aes 2,22 Integrating both sides, 72 — 7 log (7z + 22) = x + cy or 142 —9 log (Tz + 22) = 49x 4 49e, or = 14(2e + By) ~ Dog (Ide + Dy + 22) = 4B +c or 21(2y —x) = 9 log (14x + 2ly + 22) =c or ay —2)- Blog (14s + 21y +22) =F ea A which is the required solution. 84 GOLDEN DIFFERENTIAL EQUATIONS Example 2, Solve : dy _ xt+2yt1 a ay Oe Rhea (Gi) (Ox 2y +5) Goae-y +3 (ii) (2x — dy + 8) dy = (x -2y +H de (iv) (2x + y+ 1) dx + (4x + 2y- dy =0. Gh dy _ (e+2y)41 . Sol. () Given equation is P= a vali) Put x + 2y =z, then Separating the variables Be de =de 42+5)829 31} lia Berd al dz=de vr [3-4 zal 2 Integrating both sides, peed log (42 + 5) = +0, or 4 + Ing (42 + 5) = Rx + Be, or faut 2b) + log (dx + By +5) = Br ee or PYy —2, + log (4x 4 By +5) <0 is the required solutio: . a 5 dy x=yt3 7 (ii) Given equation is dx" Wan yy8 sal) Putx—y=2,then dy _de dy ac 1-2-5 BaF . 4g dt 243, dz, 243 | 242 Brom Fm ae eee de 1 B25 2248 Separating the variables, 248 ae ade or (+ 4j) ana 242 re2 Integrating both sides Qe logic +2=xse or xy) + log ey + exec or x—2y + log (x -y +2) = c is the required solution. (iii) Given equation is 4 i) SOLUTION OF DIFFERENTIAL EQUATIONS OF THE FIRST ORDER AND FIRST DEGREE 85 dy dz Putx~2y=2z,then 1-2 90 = 7 or dy de . 1(/,_dz :. From (i), a(-€) or & dx Separating the variables, (22 4.8) dz Integrating, 2 +he or (r= Dy? + 5x = 2y or at dey + dy? + Ge — 10y which is the required solution. (iv) Please try yourself. (Ans. x + 2y + log (2r + ~ Ds] Example 3. Solve : (2244) dy aty-2) de Sol. Put r4#y =2 & de ty ae then 1+ ae ae = ae del ‘The given equation becomes 2-1)(a 27 S-1 (Fle dz = 5-- 1+ dx G+DG-D 2 +2-2 dz 22-2-2 227-4 = za? 2-4 de Fe e-2* e228 * dz = 2dx = {1+ ar ezae 22 ‘i 1 Integrating z + 3! dz=2+e 3 2+ F logis! -2=2c+e 2=2 1 1 = xtyt sy log lize yP-2)=2r+0 =z log lix+y#-2]=x-y te. Example 4. Solve the following differential equations : bey (ii) (x + yd - dy) = de + dy. 86 GOLDEN DIFFERENTIAL EQUATIONS Sol. (i) The given equation is dy (x4y47 de” D4 +3 ~@ Put x +y =2 80 that 4%. From (1), ——1=——— or or M8 ads or [2 a |eceae 3 82+ 10) Integrating Footbag a2 + 10)= 2 40° or Glx + y) — 11 log [B(x + y) + 10] = 9x + Se" or H2y —2)— 11 log (Br + 3y+10)=c where c= (éi) The given equation is (e+ yWide —dy) = dx 4 dy - dy _xty-1 or Gty-DdeaaryeDdy or ee Put x + y =z and proceed further yourself, Ans. (y — x) + log (x + y) +c) TYPE IV. LINEAR DIFFERENTIAL EQUATION Def. A linear differential equation is that in which the dependent variable and its co-efficient occur only in the first degree and are not multiplied tometber. ‘Thus, the standard form ofa linear differential equation ofthe frst orderis & + Py= Q, where P and Q are functions of x or constants (i.e, independent of y). Art. To solve the equation 3 + Py =Q, where P and Q are functions of x only ‘The given equation is +Py=Q ‘Multiplying throughout by e!?#", we get dy dy Pde JPde 2g, iP 3 a? +Py.e Q.e WD @ py lPdey_ Y iPa ff iPae- Now a a ty gle ] Hse yy cite 7 Upael [> Lfemae Zine] SOLUTION OF DIFFERENTIAL EQUATIONS OF THE FIRST ORDER AND FIRST DEGREE 87 ds =X hay 2 From(, < ly. el P4y=Q. of PX Integrating both sides w.r.t. x, we have yeh afQ.c PH dete which is the required solution of the given linear differential equaiion. 4 JPde po OY ofp. [Pde elPte pa OY ef Paes By, e' Note 1. The factor e!P¢*, on multiplying by which the L.H.S. of the differential equation be- comes the differential co-efficient of some function of and y, is called an integrating factor of the differential equation and is shortly written as LF. Note 2. The solution of the Linear equation {Y. + Py = @, where P and @ are functions of a x only, is YLF) = /QULF) dx +e | Remember ‘Note 3. Sometimes a differential equation becomes linear if we take y as the independent vari- able and x as dependent variable. In that case, the equation can be put in the form z +Px=Q, where P and @ are functions of y (and not of x) or constants. LE, (in this case) = e! P4¥, and the solution is X(LF.) = /Q. (LF.) dy +c. | Remember Note 4. While evaluating the LF., it is very useful to remember that ef © = f(x). Thus ebh* xt Note 5. The co-efficient of 2%. it not unity, must be made unity by dividing throughout by it. Example 1. Solve the following : 42) B 42a nae Gi) & ey ecx stan dy 9 _, ay (ii) FX awe (iv) © ay tan x-2 sin x. P ds x dx ~? Sol. (i) Given equation is+2 2 420 = dx? Dividing throughout by 1422, (10 make the co-efficient of & unit dy, Tt is of the form Here 88 GOLDEN DIFFERENTIAL EQUATIONS, oe “ UPnP cel Fea Hence the solution is meee aye yP)e J Q.(F)dx +e 4x7 or Miex)=f ja dsehdree or WL +z%)= J dtd +e 3 or Heat) = AE 46, (i) Given equation is x +(seex).y=tane It is of the form f spyee Here Pssecx, Q=tanx ‘ LB. = of Pee a gloeexde _ glosieees+ tans) «soc e 4 tan x Hence the solution is y-GF)= f QF dr+e or y (seca +tanx)=[ tans (see x +tanx)dx+e = secxtanxdr+ [ tantrdr +e ssccx+ | (sec?x= I) de+ensccx +tanx-x4e. Another Form of Above Question Solve oss % + y mains [Hint. Divide throughout by cos x] ii) Given equation is Beha Itis of the form Bsry-@ Here Hence the solution is WLF)= J QUF)de+e “ or yea ft oxdese or watte SOLUTION OF DIFFERENTIAL EQUATIONS OF THE FIRST ORDER AND FIRST DEGREE 89 oa con ig AY. ‘ (iv) Given equation is ae (tan x).y =-2sin x It is of the form % +Py=Q Here P=-tanz,Q=-2sinx LP. = ¢! Pat gg Stns zel®O"™ = cos x Hence the solution is yE)= J Q.LF) dr +c or yoosx= | -2sinxcosxdr+e 2-f sin2ede ses or ycosx =} c0s 2 +0. Example 2. Solve the following : @)secx & = in & + y tone ees i) see x Fe ny tain iid Ge ty tan x= see a dy , dy . Mit) (1.422) & + Bay = cos.x Civ) log a + y= 2dog x (Delhi, 1996) (Meerut, 1998) dy xtytl © Ox esl Sol (é) Given equation is sec. 2% ~y=sinx dy Dividing throughout by sec x, to make the co-efficient of ” unity, dy . ae 7 (6082) -9 = sin x cos x It is of the form # +Py=Q Here Pa—cosx, Q= sin xcos x o LF.= el Pde = ef mexdr serine Hence the solution is y- GF) /Q.0F)dr+e or y.e-tin« = sin x coax. e-** dx +c=) tet dt +e, where f = sin x et * = at t 5-J 1.2 dt+es-tet-etee sse(t+Deee—e* (sings Dte or ys-(ing+ lees, 90 GOLDEN DIFFERENTIAL EQUATIONS 1) Gi ion is Gi) Given equation is Gy + (tan 2)..y = see x It is of the form & srye@ Here Patans; Q=secx * LP. = el Pat gltinzde 5 clones 5 sec x Hence the solution is. y ALE) JQ (LF) dete or ysecx =[secx.secxdx+c=]sectxdx +e or yeeex=tanz+c or = - y= sin +c cos. (ii) Given equation is 1 + 24) & + dey = cose : Dividing throughout by (1 +24) to make the co-efficient of & unity, It is of the form Here s L Hence the solution is y GF) =[QUF)de+e 1 lea? sfoosrdxte or M1+2)ssinz+c. or 1 +22) = f cos x. (14+3%)dx 40 (iv) Given equation is x log x x ty = 2loge Dividing throughout by x log x to make the co-efficient of # unity, It is of the form Here 1 Ve ag n LP = elPe oo! Fhe ol ie week: o log Hence the solution is y CLF) =/Q. (LF) dr+e SOLUTION OF DIFFERENTIAL EQUATIONS OF THE FIRST ORDER AND FIRST DEGREE OL or vlogs f 2 logs de +e 1 (og x) * Mtflay fards or ylogx=2f 2 togede+e=2. °E +6 lia net n or vlog x = (log 2)? +e. . . dy xtlty 1, ¥ (v) Given equation is de xtl alt | Note ay 1 *" drei 77! It is of the form BrPy=Q 1 Here P=-—— x+1 » Qt 1 cee dP GTM gett gages 1 = oe Gree Hence the solution is y-(LF)=/QALF) dese 1 1 or ysl sure or slog e+ D+e. xed Example 3. Solve the following : P dy wt ” dy G4) Fe -nyz eee (ii) cos? x Fe +y = fanz. Sol. (i) Given equation ist +1) & — ny mete + aye Dividing throughout by (x + 1) to make of the co-eft of 2% unity, dy on de ee TP . ® rye It is of the form Ge tPy=Q n Here Peo) Qeetic +r 1 anf tude -- eiPar_ Tai wer MiadeD _ glomtix+ D “ IF. =ix+1y*= 1 (e+ 0" GOLDEN DIFFERENTIAL EQUATIONS Hence the solution is 1 epee wt eee fee" ies" sfedxte or yates DMe'+e) (i) Given equation is cos? x & +ystans Dividing throughout by cos? x to make the co-eff, of & unity, + (sce? x) .y = sec? x tan x Iti of the form ® .pyeg Here Pesec?x, Q=secxtanx LB, = el Pat 2 glee eds < ptane Hence the solution is y etme sf soe? x tanec! de te =[te'dt +c, where t = tan.x ste'-+enet(t—l+e =e (tan x—1)+¢ or ystanx=14 ce Another Form of Above Question cos! x. # 4 ycosx=sinx Example 4. Solve the following : ws # . ia aap given that y = 0, when x= 1 x Gi) (1 +23) % — oe ty stants Gi) « +2y tanx=sin x given that y = 0, when x =~ 3 ae 1 dy _ = hea? aa (o)x Se + By —a? log x = 0. Sol. i) Given equation is % —2 1 ae P41?" GP DF iv) Y It is of the form LB sryeg 1. (x? + a a J tei* Q= te we eheeteD x41 SOLUTION OF DIFFERENTIAL EQUATIONS OF THE FIRST ORDER AND FIRST DEGREE 93 Hence the solution is -G2+Idree 1 ited soa5 “ a+b? or wa?4 1) stan x4 when rely=0 = tan x ‘ “ Ostamtite or Ostte 4 can Required solution is y(x24 1) = tan! x — — _ dy (ii) Given equation is (1 + x2) ea +y =tan! x, Dividing both sides by (1 + x?) to make co-eff. of 2 unity wed tan? x dy “1 4x* lex? It is of the form py 2Q 1 tan! x Here Pea @ ae 1 alii 1 tenet ag! Tee a qian Hence the solution is =] te’ dt +c, where ¢ = tan"! s: stel-le+e=elt-D+e=e™ (tant +e or ystan"!x—1+cetan's, a (ii) Given equation is S +(2 tan x).y =sin x . d Itis ofthe form SY +Py=Q Here P=2tanx, Q=sinx a LF. = el Pa wg gittanade 5 2 ltweer Soler. goc2 x Hence the solution is y.sec?x =f sin x sec? xdx +c secxtanxdr+c=seex+e or y=cos x 4c cos? x wi) Now whens = 5 .y=0 =cos 7 ae etyet Oncos 5 teen? 5 or On te. 4 GOLDEN DIFFERENTIAL EQUATIONS s ca-2 Hence the required solution is y = 00s x - 2 cos x. ax 1 +i” #) Ch: aos dy ) Gin ti a (ic) Given equation is rn It is of the form & +Py=Q Here = P= —#*_ gz 1 e418" Fy DF ae 2 ae n LR ae!Pt IHS = etlonta? +) = ebetet® (94 yy Hence the solution is 1 sete vs | aie ots Dare or yet + 1) x tantese. (0) Given equation isx © + 2y 224 tog x Dividing throughout by x to make the oo-eff. of unity oy 2 det 7 =F logx It is of the fe x is ofthe form FY + Py=Q Here p22 x * LF. = Hence the solution is ge? = Jz log x.a?dxtc= [x3 logxdr+c ‘ ‘ 4 4 xt op x x = log x. =~ J gop drtes S bogx- 7 se 16 xt x! or y= “P logs Te +e. Example 5. Solve the following : d @xa-0 oa -(e-Qy=2r-) (ii) & +y tana =x" cos x tity & = ape? 29) dy (Go) Fe + ¥ 008 x sin 2s. Sol. (i) Given equation is dy xx= 1) = = 2 = x 2x - (x = Dy = x: bv kouunion (OF DIFFERENTIAL EQUATIONS OF THE FIRST ORDER AND FIRST DEGREE 95 jor i dy It is of the form z 2-2 Here P=- ae-D LF. = e! Pa e712 logs log (r= 11 gee x7 — bog (DI fay 4 2c) fey" re * x=1 2. The solution is - 2 - j ye 2a 27) gy eu lQe-Ideseext—xee or ya -Dar?-x40). 4) Given equation io & + tan 2).9 = 2" eons It is of the form 2.9 Here Petanx, Q=2" cos ' te LF. = e/Par pftansdr - e6** «see x ‘The solution is, y see x = jx” cos x. secx dx +e=[x"dx+c met or yeeer= 7G te an - com i dy Gii) The given equation is Fao or ® yar yest ay Itis of the form a tPy-@ Here Ped, Qaxt ‘ EB 2 elFés el #* ae" ‘The solution is yee alate? devon df ste tedree | Note =} Stel dt +c, where t =a? or or or or or GOLDEN DIFFERENTIAL EQUATIONS =tet-D+e 1 1 2 Pepe atte or yas ire". ye (iv) The given equation is % + cos2.y sein 2 dx It is of the form # yg Here Pacosx, Q=sin 2x a LF, = el 4 a etla = =. The solution is y eM 7 sf sin Qe. eM dx =e =J2sin x cos xe" ‘dr+c=2) te dt +c, wheret =2tt-Dec y 28 £2 2etin x (sine — 40 y =24ein x— 1) +ce-8" 4, Example 6. Solve the following : (i) sin x 2 +y cos x = 2 sin® x cos.x ii) (27 - 1) a + 2x + Qy = BWe+ 1) ds (iii) (1-32) on +o2y=ar (iv) x= & yn ste wxd-2) 2 4 ae-by=2. Sol. (i) The given equation is sins ® « y con = 2sin? x. cons & scotx.y=2sinx ovo | Note It is of the form & +Py=Q Here Pscotx, Q=2sinxcosx LP, oe! 2 eltens cgin x «. The sotution is y-sinx =J2sinxcosx.sin xde+c=2 sin? x.cosxdr+e ysinz= 4 sinx+e. (ii) The given equation is wy B42 +2)y = Ae +1) dy | Ux+2) 0 2 dx * (oy 7 F-1 It is of the form Sy py=Q | Note Here SOLUTION OF DIFFERENTIAL EQUATIONS OF THE FIRST ORDER AND FIRST DEGREE 97 a] mth Br eh af 2. LF. = gipar. tlie tna =e"! fa 41 ar Bb (E= Ne bg iret tre . bog -)* nelle“ Wi -dagieey 2g rer =D (x+D «The solution is a 3 ySe 1 2 =I dxt+e Yen =) =< xl (e-v? xi -2x+1 a2 f iP devenaf =i" arse 4 e2f(s-aeAjaree “bf G-¥ =2[F-seedtagicr| ve xt or yx 17 = (x + Dla? - Gr + 8 log x + D teh. (iit) The given equation is (1 —x*) . * +ay nar dy, dx It is of the form Here Po—>z | Note ‘GOLDEN DIFFERENTIAL EQUATIONS (iv) The given equation is ste) @ ye eee de It is of the form 2 rwy-Q Here Pees: Q=xx-) og en loge 1-log2) 2 gles log x=) 2 Henee the solution is x = age) seep dese 3 x x inf BdrtcwZ te or yin] tates Ste 4v) Dividing by a(1 —2%) to make the co-efficient of 2 unity, the given equation becomes dy | 2x?~1 dx * =a) It is of the form ® sry=Q Here Pe cacy 2x7 -1 1 1 1 . . Now Peyi-a+s x @i-») Riry _—_‘|Partial fractions] 1 s J Pade = —tog 2 — 3 tog (1-2) 2 tog (1 +2) =~ log bx (1-2)? 4 2) = log (x 1-27 ]=log @ 1-2? 7 Jog (x (1-24 4 = 1 * LF= ee. ‘The solution is ye se) Fa ee xVi-x 1-2" fi-a* SOLUTION OF DIFFERENTIAL EQUATIONS OF THE FIRST ORDER AND FIRST DEGREE 99 Serta d (1-2) Carder 2)-v8 sj} er te “2 => Z = 1 +c => xji-x? i-e? which is the required solution. Example 7. Integrate (1 + 2°) . 2 + 2xy -— de? = 0 and obtain the equation of the eubie curve satisfying this equation and passing through the origin. Sol. Proceeding as in Example 1 (i), the solution is 4x* sexe +0 As it passes through the origin (0,0) +. ¢=0 Hence, the equation of the curve is 4x3 yl+x= 7 oO Sy +x) = 4x3 Example 8. Solve the following : wat en. B ayt-29+ 2 begs (icone 2 +ysin set dy 2. (iti) Get vs sins. Sol. (i) The given equation is sate Baya -2 4s log or It is of the form. Here 2-1 a 2-1 wp. dae, Hattie soe 1 alien where 2? = ¢ 100 GOLDEN DIFFERENTIAL EQUATIONS The solution is +1 =f hex eet x?+1 xe +1 = dete or or or (ii) The given equation is w = dy cose oe tysinx= 1 or Gy Pty = Beer Here LP. = bre Samer see x ‘The solution is y . sec or xy see (iii) Given equation is dy 2,2. .y sain 2 Le cel 2 thera gle ot «. The solution is gy tefsine dese =a (008 x)— J 2x (- cos 2) dx +e sx? cos x4 Ax sinx—J 1. sinxdr]+c or sty 2— xt cos. + 2x sin x +2 cose +e. Example 9. Solve the following : ety \de dy 1-2 i —=1 ii) el of EES wat y (iii) y dx — x dy + log x dx =0 (ivy & 4 2y cot = 32% cance! 2 in on 2 iy y (o)sin 2x Go sy + tanx i) Ge * Gods 7! ve (vii) de + 2st ds = se ds. Sol. (i) The given equation is -2e or dy 1 ie SOLUTION OF DIFFERENTIAL EQUATIONS OF THE FIRST ORDER AND FIRST DEGREE 101 Itis ofthe form & + Py =Q 1 Here Pay, Q= L : has peu a tre ee, fhe ee + Hence the solution is ~ 2 rc yal = 0 deec fe i 1 or y ef qe teten Bl te. (ii) The given equation is d 1-2x zt wget j dy It is of the form S* + Py=Q Here Ps ie .Qz1 12 “ LF.= wd (es 1 1 we ee tens | Note els gloss Henee the solution is eM a fie deteslet dere 1 1 ael43 where-—=! « —dx=dt x x or electse of alsel * or y=341 + 0c), (iii) The given equation is ydx — xdy + log x dx = 0 1 or yo Retoge-0 or 3 y. MEE It is of the form Bs PyeQ Here Pa-1, qu 8% z z ae a erbex lee’ _ Ea poe xty? 2849 = x LZ ong #9] ~? pod xp D=ME4 Ge THF OTION TT get exfopt Px wo zag Top OFM 0 = ap x Bo] + Kpx— xpf sj uorEnbe woats ou], (731) Gn + DEK 40 worpse wo Ob gq ean EE 20 neat mPa? T a +22 pape 4 psp enoue & sy WoRNJOs ayy doug aq? = gx te?" xp? = SHON | sett ag EO a=at , * axon ze-1 7 = hat FH ay om Jost » 3 sr worgumbs waar OtLL (1) x . soe ap geoesp aL fue « 30 a a 2+ aP aya? aol ae & sy woRMos ayy a9u9Fy = 4 P= al arf tay thea are ape? T aha + FE mae omaso str Tot BHMOAA LSYLL CNV YACYO LEW SHL JO SNOLLWNDA TVLINAWALIG 4O NOLLATOS SOLUTION OF DIFFERENTIAL EQUATIONS OF THE FIRST ORDER AND FIRST DEGREE 103 1 P2—+—, gsi- where Gondg' Gin 1 1a ped Far ve wm Ate 1-t a 1+ Vx +. The solution is_y. Ee fa-€ eta : =] 4 feldesenes 2 tee, at 2 (vii) The given equation is at Qs. t = se It is of the form # +Pt=Q where P, Q are funetions of s only. Here P=2s,Q = se LF. = ef? ef “The solution is = te =f 9°" e de+e y_x+yi-x? * nF Example 10. (i) Solve : (ii) For the equation x? : + 2xy = 1, show that every solution tends to zero as x tends toe, Sol. (i) The given equation is of the form dy de tPY=Q where P, Q are functions of x only. Here * =x? 1 aoe S wed a Put x = sin 8, so that dx = cos @.d8 cos 0.8 Jace eI J sec? odo tan 0 = £ =n LF.ceh™ 104 GOLDEN DIFFERENTIAL EQUATIONS +. The solution is jee j x+jl-2 eee us ye a-2? . Put x = sin 0 80 that dr = cos 8 d@ : pee a ae? rine sin 8+.€08 8 end cos @ = c08 640 ae af MH840050 pune _1 gy cos 8 cos? @ = J tano+ De"? sec? 0 d0 ef @rikede wheres =tano eG De~J Lede =(e+ Dee ace =tan@, i"? * tee -. The solution is foe or Here P= fe LF.= ‘The solution is 1 Pal = yte| atdree 1 or yteaxte or yest Asx, y 0 Hence every solution tends to zero as x — ©. SOLUTION OF DIFFERENTIAL EQUATIONS OF THE FIRST ORDER AND FIRST DEGREE 105 dx Equations of the Form G+ Px = Q where P and Q are functions of y only. Example 11. Solve the following : @ +29) * =y Gi) (1 + 97)d = (tan y — dy (Delhi, 1999 ; Calcutta, 1996) a dy (iia) (1 + 52) + Gee" y@eo (iv) x— 1054) & +9 =0. Sol. (i) The given equation is (x + 2y3) # y or 9 Fax. 28 ao «CLD, It is of the form & 4 Pe=Q dy i 2 Here Pe- ys Q=% 1 tre de comer etertayt ed y «. The solution is 1 ste] wideres| ware x x or yamre or xayS toy. Gi) The given equation is (1 + y?)dx = (tan! y =x) dy 8% tant or a+) dy 7am yer tant y xe or +9? It is of the form 2 + Px =Q dy 1 ured Ppt ty + The solution is 1 tan ty ag! tal y tant y anya fe et dye = J te! dt +c, where ¢ = tan1y aet-Dte set"? (tant y - Ve or eatanty—Lece ry, 106 (GOLDEN DIFFERENTIAL EQUATIONS ii) The given equation is a, tan ty) SS es (149904 (ee) 0 dx tan“ y 149) S ¢x-e' =0 C we t* d& 1 eumty or dy Sing? 8" Tey? or Itis of the form * 4 Px =Q dy 1 dy red BF =. The solution is 1 tant 5 = xemmty sf £ -e™ "Fay genle!efdt+c wheret=tanty ley? sletdtsenh dre or red MOY ge, (io) The given equation is (2s — 10y*) 2 +y =0 or 9 $422 1098-0 or 8 2 ergy dy y¥ Wis ofthe form S +P =Q 2 Up a Pte IY ther ot = yt +. The solution is ay? = [l0y?. yady te = 10Jytdy +e ae penaytee Example 12. Solve the following differential equations : We de+(1+xe) dy=0 (ii) cosh x dy + (y + cosh x) sinh x dx = 0 sy dy - iv) sing 43y2 Gi) Fe +y cot x = 2 cos iv) sin x SE + 3y = cos x (v) (x + log y) dy + y dx =0. Sol. (i) The given equation can be written as dx or Sisal = -e ogy tits 0 e S| + x a SOLUTION OF DIFFERENTIAL EQUATIONS OF THE FIRST ORDER AND FIRST DEGREE 107 ; dx It is of the form dy +PxsQ where Pel, Q=-e LF. = e!Pd oo «. The solution is x. -e* .edyte or xe=-]dy+e or xe’ty=c. Gi) The given equation can be written as cou x & ¢ ysinh x4 cosh xsinh x= 0 dy dr Ikis of the form 2 Q is of the form © + Py = where Pstanhz, Q=-sinhx LF. = of PY mp gltanhade we plaeoth = oogh x .. The solution is or +y tanh x=—sinhx yooh x= Jrsinh xeosh x dx +e =-3] sinh 2x dr +e or yooh r=~4 cosh 2e4 6. (iii) Please try yourself. (Ana. y sin x =~ } cos 2x +e] (iv) The given equation ean be written as dy oe 1 BY cosee x = cot x It is of the form & +Py=Q where P = 3 cosee x, Q = cot x state _ mele LF. = ef Pes a ofBemerds = ¢ stan* > J aa-). 2% +6 =f Sef ase f -42- 2 Jdtee +1 PL 2 Gru Ptanltse sto lit ZL or ytan? 5 =~ tan? 5 + 2tan > -x +e. (v) The given equation can be written as sa logy ty F #0 or It is of the form & + Px =Q dy where pat, gq=- ee y ¥ 1 LP =elP& J 7” goer = The solution is logy ve] ~ 7 vdyee =-J (log y). 1 dy sere [omg y.o-f tra] ae c or wye-ylgytyte or xe d-logy +5. Example 13. Solve the following differential equations : ort —y = 2x? cosec 2x (Kanpur, 1996) G+ @ ay neers Gil) (1 ty + xy) dx + e+ dy =0 (iv) sec x dy = (y + sin x) dx. Sol. (i) The given equation is Rie nt ¥ P xB -yn2etcoseeds or SOLUTION OF DIFFERENTIAL EQUATIONS OF THE FIRST ORDER AND FIRST DEGREE It is of the form SY +Py=Q where p--2, Q=2r cosee 22 : IR ce?! oe * sere . The solution is tex ax) 4 de yg cosec 2x) 5 dr +e tal =2f covec De de be = 2 x ELD | or y =x (log tanx +c). (ii) The given equation is a +9 2 tyee™ or Itisofthe form & + Py=Q gant where : 14x" Lea? L atyar IF.=elPH es J TT 2 gia The solution is ean tan"? ten" ve =] e"* dese * 1+x? =f tdtse=} +e wheret =e * or ye red ttm see or ya pes tce rs, (iii) The given equation is (Lay +22y) de+(e+x)dy=0 or ses) yya4e=-1 110 GOLDEN DIFFERENTIAL EQUATIONS .. The solution is - de vive wae th tee J pte or xy + tan sc (iv) The given equation is see x dy = (y +sinx) de or ® ccoexy + 5in 2 or oY _ ycos.x = sin x c08 x dy POR d It is ofthe form © +Py=Q where =. P=—cos. x, Q=sin x cos x LB, = el Pat a gloria a maine -. The solution is yiette| dnzeonretedese = J tetdt+e, where t = sinx etceo-[ 1.Cendree antet-efsen-(t4Detee or yee easing Desc or == (sin x4 1) + cetins, Example 14. Obtain the equation of the curve whose slope at any point is equal to y + 2x and which passes through the origin. Sol. Since the slope of a curve at any point is & . .. The differential equation of the curve is dy dy grt or Bayar It is of the form # +Py=Q where P=-1, Q=2 LF. =e! Pa = el ae" «. The solution is gers] aretdr+c =2 [zeen-J Le") de] se22 eter} ee SOLUTION OF DIFFERENTIAL EQUATIONS OF THE FIRST ORDER AND FIRST DEGREE a or yro2r=24 0% =D which is the equation of the family of curves, If it passes through the origin (0, 0), then O=-24¢ or cH2 The required particular member of the family is Qe tyt 2am 2e%, Example 15. Solve the following differential equations : (i) & aye Get (ii) 2y' +4y sx? -x Gili) y’ ~ 2y = cos 3x (iv)y’ +y =sin x + cos x. Sol. (i) Hint. LF. = e& The solution is y.c=f 6et.c%dr+en6] dese or ye™e6, or y= Bet + ce, Gi) Hint. LF. = e™* The solution is 1 yee | Fu2wetdere oh [oe Ef oe a TH +e 1 1 et geen] Ge ex ot wet f Ge De dere 1 on eo (Pa) eM [tv -J2-Fa]re z a 2 Gt-aet— 5 (e- Dette 2. S40 (2x? — 2x —| ale dew* @-IP tee, or ye or ye Gii) Hint. LF. =e ‘The solution is yerten fom cos Sx dete et = Carag? F288 +8 sin Ox] +e pe (eos be+ bin v9] [- Jet 205 bx de = or y= Gh @ sin Be - 2.00832) + oc. 2 GOLDEN DIFFERENTIAL EQUATIONS iv) Bint, LP. = = The solution is ye"= J (sin x + cosadet de+c =f sinz.edr+ J conzedrec zsinx et f cos ret de + f cos ret dr +c sesine+e or yasinx+ce™. Example 16, Solve the following differential equations : x a dt elog x wy eye i) yy I+sinx (iii) ¥ +9 = Tyce Gv) xy y= —- De Sol. (i) Hint. LF. = e ‘The solution is yeref =f [Artal eaee [Form: J ifte)+ fale" a] 1s: -1 =Jajyeu-] wap ate 1 -1 -1 =yie-| eat) Gane tere ee 1 +e or ye yyy tot. (ii) Hint. LF. = e* ‘The solution is gota J (HERES) ede =f (tor+ 2) erdeve [Form: j fo +f'@he™ as] =f loge etdr+ [ tedr+e slogset-f 2eac+f rerde+e setlogrte or ynlogx tee, (iii) Hint. LF. = e* L+sins The solution is ye = | [popes ede te SOLUTION OF DIFFERENTIAL EQUATIONS OF THE FIRST ORDER AND FIRST DEGREE 13 1+2sin = cos = =f 2a dere 2.008? = 2 =f (tnd 1 ag Z)edese [Form: J i7ta)+ f'cone® dx] =f tang .erdes fF sect S dere wtan % et [ SeoctZ eras | } sect? etdrse 2 2 2 29 x x setan Fae or ystan J +e. frda (iv) Hint. LP. =e" * =— ‘The solution is - 1-3 dete [Form:f ifts)+ f'ohet ax] Lope} -1 1 sze-Ja ede+f Gidea eette or yaetece. Example 17, Solve the following differential equations : ™ , (ty tan x= Be +32 tan (i 4% aeons BE Gi) ede + fe — 7) dy =O. Sol. (i) Hint, LF, = * = g2e x The solution is ysecx =f (2x +x? tan x) seexdxr+e =f 2esccede+ f x2(soertanz)de+e =) roccede+s%svcr— | ae scexde +e sxtsccxte or y=a%4ecosx. (ii) Please try yourself. [ans y=sin x] (ii) The given equation is 9G exvreo =? + Sle ein or 14 GOLDEN DIFFERENTIAL EQUATIONS It is of the form & mre 1 here P=-, sy wi y Qzy LP. =e! P 4 obey ay «: The solution is 4 ye] y.sdy+e or gyette Example 18. Solve the following differential equations : (dr + (2r cot @ + sin 20) d0=0 (ii) e sec? y dy = dx + xdy - -) dy ¥ Gil) ye dx = (3 + Bre )dy io) Fe" Bylogys y= (v) J1-y? de = (sin! y—x) dy twit Base aay 64 Sol. (i) The given equation ean be written as dr . ag t (2 cot Or =— sin 26 It is of the form * +Pr = Q, where P and Q are functions of @ only. Lp ae! 280% _ atesine _ chess" sin? 9 . The solution is r.sin?@ = J —sin 20. sin? 6 d0 +c n-2 f sin? 0 cos 0d0+¢=- or 2rsin?6+sin‘@=C where C = 2c. Gi) The given equation can be written as Facer eaty tree oe ‘The solution is xia] ereccty.ody+e = J sty dy restanyte or zee? (any +e). (iii) The given equation can be written as per nyt Dee SOLUTION OF DIFFERENTIAL EQUATIONS OF THE FIRST ORDER AND FIRST DEGREE Ws Dividing by eto make the co-efficient of SE unity, we have Geyer dy y It is of the form = + Px = Q, where P and Q are functions of y only. 2 ned? they geet -. The solution is 1 1 z.age] Fer. Gadyse ¥ Jv ye or . 2-J dyton-eF+e or zay2(e-%) (iv) The given equation can be written as dx _ 2ylogy+y—% x Sn SUEY T YN * ology +1-— ay y logy y or ot st otogy 4 dy y 1 ty ined? =eay 2 The solution is ays J Qlogy+ Dydy+c =2 J dogy).ydy+] ydyec 2 2 x ly = 2,006 9 5 J 12a]+f ydy+e =F logy- J ydy+ f ydy+enytlogy te or xeylogy+ = (v) The given equation can be written as or 16 GOLDEN DIFFERENTIAL EQUATIONS sin ty aie! vi-3? =t-Deltes(sin-ly De 740 The solution is x. eM"? = [ vdyse=| tetdt+ewheet=sinty or x=sin?y—1+ ce", (iv) The given equation can be written as & 2 1 Get ya oee Proceed further yourself. Tans. y 224214 03] ‘TYPE V. BERNOULLIS EQUATION ‘To solve the equation SY + Py = Qy", where P, Q are functions of x only. The given equation is +Py=Q" onl) Dividing both sides of (i) by y", to make the R.HLS. a functions of x only. y 2 +P ag i) Put y™ =z, then an). % BR a& a .X4m. I-n ar *FET@ + (ii) becomes or Sea-m.Pesa-me which is a linear equation in 2 and can be solved. In the solution, putting 2 =, we get the reqd. solution, Trample 1 Sle te flowing: ay yy ne .%,v of 24 zey @2atet ye (iy Ba 2895 ay (iv) (y log x - 1) y dx = xdy. Bol. (i) The given equation is 2 ¢ 2 252 Dividing beth sides by », we have yt Bayt tan od) Put ‘=z, then -p tie SOLUTION OF DIFFERENTIAL EQUATIONS OF THE FIRST ORDER AND FIRST DEGREE. 17 1 1 + The solution is 2,2=[ -de+e x x or yt 2 entogs+e lvzeyt or a atgzec. x it) The gi ion ise YY (di) Tho given equation is 2. 5° 7 = 2. Dividing throughout by 9? w yt , wr ts Ai) ay de - = Put yl=z, then ee ++ @) becomes de 1d & 1 1 Fae ae Be BF which is linear in z. 1 1 Page Qn oF aac lige La wed FY gt 2 cot F a Le 1 +The solution is z. VE = J - 59 ve drec or pie} f race or or xayl+evz). Gié) The given equation is oy ae Dividing throughout by 9° yi igtes Put ytez, then - ays PP my ax 118 GOLDEN DIFFERENTIAL EQUATIONS ~. (i) becomes: lide dz pate org bee? which is linear in z, P=-2r,Q=-2x5 LF, = el 2dr The solution is zie aJm-ate drec —2e.x8 eo de+e=s—teldt+e wheret=—x? s-e(t-D+c or yret s-e" Gxt-ee or yeexttlece™, Gv) The given equation is (y log x - 1) y dx =x dy dy oe dy a log Dy? loge - as yay? SBE or x Paytloge-y or B42 yay? 8 Dividing throughout by 92, dy 1 Jog x 2 slik tgs wf) which is linear in + Thesolutionig z,4=f —ME% lave SE xo‘ 1 -te-f loge. Sar or ys J tex. Sarre 1 Vp 4 2-| hog x.(-27)-f 2{-+ " an [er (3-4) 1 1 1 or 4 2-|-Fhog e+ f Sax xy [ x 8* Iz |e or j--[-dues-2] +e or A a logx +1 4ce. xy x x, x SOLUTION OF DIFFERENTIAL EQUATIONS OF THE FIRST ORDER AND FIRST DEGREE 19 Example 2. Solve the following : dy dy te i) =y? tan? in ® 412 & -aytan x =y? tan! x Eres (Kerala, 2001) dy 2 xt dy * yex, wins Bea ? oy Re ager. Sol. (i) The given equation is & _oytanz ey? tants Dividing both sides by 2 yt, © ay tans atants i) dy _dz ce ay? Put y =z, then woe de From (i), ~ 2 -2etan 2 = tants or & s2tans.22-tante which is linear inz. P= 2tanx, Q=~tan*x. ER, = ef 2imrde _ gles recs _ glosses _ sae? y >] -.Q=754 ay veoh SET gee cog +. The solution is raendel Sy er Ddescore a+ Dace, (ii) The given equation is oy 2 & -ytanz =—y? seca 136 GOLDEN DIFFERENTIAL EQUATIONS 1 ai. or ~. Ds =tane=seex A) yd y apt 1 dy _dz Putting | = < s0 that Fae dz + Equation (1) becomes 7. +7 tan x= secx whichis linearinzwith © P=tanx, Qeseer LP we! sel taneds _ glorwecs oy <. The solution is z.seex= J secx.seexds +e 1. or J sce xs tan r+ecor) =sinx+ccosx, y y (iii) The given equation is 1 ogi 2 de ty 28iny cosy =x cos? y Dividing by cos? y dy | 2tany sooty Fa eat Al) . dy _ dz Putting tan y =z so that sec?y ae = ae dz 2 + Equation (1) becomes + Dz = a4 which is linear in z with p=2, Qex8 a * Lea elPHtae) «Y aetier acter a2 2. The solution is ’ ete J A .tdescorstanys = +e. (iv) The given equation is dy _y__ yoose dee ww) ds Puttingy*=2 sothat ~ 94 5 = cop BIR Se o pe SOLUTION OF DIFFERENTIAL EQUATIONS OF THE FIRST ORDER AND FIRST DEGREE 137 1 __ cose x 3 = Equation (1) becomes - x or or y3PeB8sinxee or 4 -Beine +e. Example 12. Solve the following differential equations : pee yt, 2 Wy gaat (i) tee d (iy ® sin xy con y= 0 (iv) sec? y SE 4 Bx tan y = Sol, (i) The given equation is dy Pa ets (1) ; dy _ az dy _1dz = 2% 4 dy ia Putting y* =z so that By’ de de or Ya 73 de <. Equation (1) becomes je ares or @ ose ane which is linear in 2 with P =~ 3, Q= 3r a= vThesolutionis = z.e*=f Swdree € or Pom ear, -Ja. e* 1 ares Ste or Yerx-g +ce™, (i) Please try yourself. (iii) The given equation is cot x = —y? cose # or 4 dx yx Put 5 = 2 and proceed further yourself. 138 GOLDEN DIFFERENTIAL EQUATIONS (iv) The given equation is dy soe? y & 4 De tan y= a3 (1) i dy a Putting tan y =z so that sect y Te oe Equation (1) becomes SE 4 Bez =8 which is linear in z with P = 2x, Q=33 LF. =e" + Thesolutionis z. 0" =f te" drte or ef tany=f x.a2e dr 4c ff tidtse wherer= 22 a fet Piet ai = 5 [e'-fre dt] +025 ete +0 =} at-Der + 1 - or tany=9 G-Dece , Example 13. Solve the following differential equations : oe ay .. dy (i)sin y F* = cosy (1 =x 008 y) Gi) Prayer ds (iva & ~yseex= 93 tans. Sol. (i) The given equation is siny & cosy =~ cos? Ye iy y Dividing by cos? y 4 secytany 7 ~secy =—x Putting see y= so that secy tan y $Y = & “Equation (1) becomes e. which is linear in 2 with “The solution is z.e*= f ~rerdx+c SOLUTION OF DIFFERENTIAL EQUATIONS OF THE FIRST ORDER AND FIRST DEGREE 139 or a Gi) The given equation is > + ay = 2°y* fl) d: Dividing by yy 2 ay? Putting »~ = z, so that - . Equation (1) becomes -4 . which is linear in z with P =- 32, @ = -3x? o2s* P= elPa 6 «. The solution is 3 ate -is z 8rd. 2 desenf stare 2 dete afte =? = J -jtedtscwherees- > 2? 2 t 2 2 Ete J relat] +e-- 5 tee By : pag 3" =-2(-2 2 ) = or yee =-3 2° lle ‘The given equation is dy 2—- =y? tan. A) B —yeeex=y®tanx or Putting ~ se ne sothat 3 2 & -. Equation (1) becomes +zsecx= tant which is linear in 2 with P = see x, Q = tan x LR = of PA 2 pheno 4802) | ey tan e «. The solution is 2 (seex+tanx) =f tan x(seex + ton adder +e or =p (seerstans)=f (secxtanx+ sec?x-Ddr+e ¥ (sec x + tan x) = sec x + tanx—x+c. 140 GOLDEN DIFFERENTIAL EQUATIONS Example 14. Solve the following differential equations : Gr sin 8 d0 + (7? —2r? cos 8+ cos 8dr =0 a ye y On ne Sol. (i) The given equation can be written as ring 22 4 (1-299) 008 = =r? Dividing by r, we have 9d, (2 . sino 2 (2-2r) eoso=—r4 li) Putting cos @ = 2 so that ~sino a +. Equation (1) becomes = - 2 4 taarlan-n atl; a 1 or Fie (ve-dene Ikis ofthe form % + Pr = @ where P and @ are functions ofr only. Ser Fle es -tege 2 tog] 1 Pateg) tog ? Fae" .enr LF. =e . The solution is 2.2 gt = [r?.2e" drtes fe rdrte r 7 ob fetdtve whoree=/? or Zoos@=r(i+Ce"") where C= 2c. (ii) The given equation can be written as gue Ebel Dividing by Jz, we have $e-8-4 =D Putting JZ =2 so that nee , ait Equation (1) becomes 2G-E-t iy 75 Tess the form SE + Pz =Q, where P and Q are functions of y only. SOLUTION OF DIFFERENTIAL EQUATIONS OF THE FIRST ORDER AND FIRST DEGREE. 141 1 ~ Fy gp te we 28? cp ory K . The solution is 2. = J a7 Te dye or Fag] Sec or ve = (Joey) or fz = 9 (low Vo +e). TYPE VL EXACT DIFFERENTIAL EQUATIONS Definition. The equation Mdx + Ndy = 0 [where M, N are functions of x and_y] is said to be exact if Max + Ndy is the exact differential of a function of x and y, ie., if Mdx + Ndy = du, where w is a function of x and y. Remember. Ifu = f(x, y) be a function of two variables x and y, then the total differential of u or the exact differential of « is given by du= x ode + = dy ie. du is the sum of the two partial differentials du ye Art. To find the necessary and sufficient condition that the equation Mdx + Ndy = 0 may be exact To find the necessary condition. ‘Let the equation Mdx + Ndy = 0 be exact. ‘Then by def. Mdx + Ndy = du, where u is a function of x and y ou ae and Ma a Equating co-efficients of de, ut au aes a Equating co-efficients of dy, N= 5 OM _ ue ON Oe so that yi oe dey aa aN aa de Hence fy "ox dyax - dxdy is the required necessary condition for exactness. To prove that the condition is sufficient or ie, Given = = to prove that Mdz + Ndy =0 is exact a2 GOLDEN DIFFERENTIAL EQUATIONS Let J Middx = where integration has been performed treating y as constant a au au sothat &([Me)-E MAE . aM _ au “ ay aya . aM _ aN. Bu au Since Five) and SE 5 + Gi) becomes x . = Integrating both sides w.r.t. x, treating y as constant. ne # +a function ofy = 3 + fly) (say) From (ii) and (iv) Mas + Nay » % ac (+109) ay dex Hay) + foxy =du + fy) dy = dlu + [fyddy] which is an exact differential. Hence Mdx + Ndy = 0 is an exact differential equation. Cor. Solution of an exact differential equation. If the equation Mds + Ndy = 0 is exact then Mds + Ndy = dlu + fy) dy) S Mdr+Ndy=0 => dlu+Jfy)dy]=0 Integrating both sides, we get u+lfdy=c where c is an arbitrary constant. From (i) u= fo Mae y~censtant From (iv) fly) = terms in N not containing x. «From(vi) [ Mdx+ J (terms in N not containing 2) dy =¢ yet which is the reqd. solution. Working Rule 1. If for an equation of the form Mdx + Ndy = 0, Iwhere M, N are functions of x and y] SES, itis exact li) well) (Ett) ali) velo) | using (v) SOLUTION OF DIFFERENTIAL EQUATIONS OF THE FIRST ORDER AND FIRST DEGREE 143 2. Then the solution is J Max + f = “equation (?) is exact, Hence the solution is J) Mae+ {terms in N not containing 2) dy =¢ yeonutant ie, J Gay) de + |ydy we y-eeartant e 3 or Froese or 3 - Bary +73 = 3c or x8—Saxy4 y= A. Example 2. Solve the following : (i) sec? x tan y dx + sec? ytanxdy=0 (ii) (1 seiyars er (t-¥) dy=0 (iti) [» (1+2)+ eos | dx + [x + log x-x sin yidy =0 (iv) cos x(cos x = sin a sin y) dx + cos y(cos y — sina sin x) dy =0. Sol. (i) The given equation is _ see? x tan y dy + sec? y tanx dy =0 wali ‘Comparing it with Mdx + Ndy = 0 Here M= sec? x tany ; N =sec?y tan x me ON sec? = sec? x sec? y ; —— = sec? y sec? x oM a since 2M ON. equation (i is exact. ay * ox SOLUTION OF DIFFERENTIAL EQUATIONS OF THE FIRST ORDER AND FIRST DEGREE 145 Hence the solution is J) Mae+ (terms in N not containing x) dy =¢ y-onmunt ie, J scctetany doe yconmant or tanyJsectrdxe=c or tanytana=e. (ii) The given equation is (1 set dese (1- 2} ay 20 Ai) y Comparing it with Mdx + Ndy = 0 Here Malet, Nae! (:-2) aM x x OM ay (- 2) * gy oy ( #) x ae esheets OM -oN yy oe’ Hence the solution is Since 2 equation (i) is exact. J) Max + { (terms in N not containing x) dy = ¢ y-ecustant ey ie, J (+e) dr ac or oreoee or xeyer? xe, y~ecstant i y (iii) The given equation is [>(2+2] somes ]de tes toga —zsin hey =0 wi) Comparing it with Mdx + Ndy = 0 Here Mey(1+4) +0009, Nex -+logx—xsiny Moe Lesiny, Mate Easing . eM _ aN Sonn (th % Since, Dy 7 ae Squation (i) is exnet, Hence the solution is J Mae + terms in N not containing 2) dy = yo constant 146 GOLDEN DIFFERENTIAL EQUATIONS 1 fey J [>(1+2) +009] dene y-eveatant or 9 f (et) ae reny li dene x or ye + logs) + x 008 y = 6. (iv) The given equation is cos x (cos x — sin a sin y) dx + cos y (cos y — sina sin x) dy =0 wo) Comparing it with Mdx + Ndy = 0 Here M = cos x(cos x — sina sin y) = cos? x —cos x sina sin y and ‘N= cos y (cos y = sin a sin x) = cos* y = cos y sina sin x Since Hence the solution is J Mar + J(terms in N not containing x) dy = ¢ y-eiaaant ie, J Coste sina siny cos x) de + feos? y dy =e yoeinaas 1400820 i vgs L+singy ,_ or J : de~sinasinysine+ f “5282 dyzc eo d(> 21) or sina sinx siny + 5[¥+—>~ | =e. Example 3. Solve the following : : (ade + ydy = EAA — yee) (Kanpur, 1998) x+y? GG? o> + dx) de + (Bry 00" - By*idy =0 (ii) (2ny + ~tan y)dx + (2° —x tan? y + sec y) dy = 0 (iv) feos x tan y + cos(e + plex + [sin x see? y + coste + yidy = 0. Sol. (i) The given equation is ld) Comparing it with Mdx + Ndy = 0 SOLUTION OF DIFFERENTIAL EQUATIONS OF THE FIRST ORDER AND FIRST DEGREE 147 a®y atx Here Marto iNav aye OM yg gt StH I= Hy) _ a(x? - y*) oy it +y7? (x? + y?}? ON gga Oty) = xl2x) | (x? — 9?) ox @+yy (x+y? . aM dN by Since yim oo equation (i) is exact. Hence the solution is Mdx + J (terms in N not containing x) dy = ¢ constant : . ay ie, f(x ‘y- constant ® +y dx : ae from { 22 or # yy fast [ate asf ay ne 2 ‘here y is constant 2 or = say. Stan or x? ty? 4 2a? tant = 3 2% y or sta yPe Dat tat SA (Gi) The given equation is (2 eX + 4x3) de 4 (2xy. e®” = 3y2) dy =0 wn) Comparing it with Mdx + Ndy = 0 Here May%ee? 4 4x3; N= day e9” — 39? aM a 2 aN 2 5 ay By eV ty. oe" Bey BE = Bye + Baye? wv ay. e #209. 08” = dy. 07) Baye?" Sin 2M aN equation (i) is exact. : ce yt ae ion (i) is exact. Hence the solution is J) Max + {terms in N not containing x )dy =¢ y-coeiast ie, J oer’ + 4st des | OByidy =e y-eonstant 148 GOLDEN DIFFERENTIAL EQUATIONS: sos or Ff tae gh o, 4 3 yeaa or PE atv neon et axtayne om er a o y-oomstant [ Nowe. Jorden. (iii) The given equation is (2ry + y— tan y)dx + (x? x tan? y + sec? ydy = 0 wali Comparing it with Mdr + Ndy = 0 Here Ma 2ey +y—tany ;N=x?—2 tan?y + sec? y am an om. - jon noe tan? Gy 7 Bet La necty | 5 = Be tanty = 2x —(sec? y ~ 1) = 2x =tan?y aM al Since ee. +. equation (i) is exact. Hance the solution is J Mc + J (terms in N not containing x} dy = y~ constant ie, J Bayt y—tan yids +! sec? yay =e tant 2 or Dy > tyz—xtany+tany sc oy xy +2y—xtany +tany =e. (iv) The given equation is (cos x tan y + cos (x +y)idx + [sin x sec? y + cos (x + ykdy =0 lB) Comparing it with Md + Ndy = 0 Here, M-=cosztany + cos (x +5); Nw sin x sec? y +.cos (x+y) aM Sy momsz nec! y sin te y) x = cos x sec? y — gin (x + y) oM Since ss «. equation (i) is exact. Hence the solution is J Max + terms in N not containing #) dy = ¢ y= dosent SOLUTION OF DIFFERENTIAL EQUATIONS OF THE FIRST ORDER AND FIRST DEGREE 149 ive, J teosxtany +coste+y)l de=c a-erat or tany sinx +sin(x+y)=c. Example 4. Solve the following : (iy sin 2x dx - (y? + cos? x) dy = 0 Gi) (a4 = 2xy? + y) dx = (2x"y = dry" + sin yidy = 0 (iii) (22 — Bey + By2)de + (Ay? + Bry —22)dy = 0 (iv) (2 + y? + etde + 2xy dy = 0 (v) (222y + 423 = 12xy? + By? x0" + ody + (12x%y + 2ay? + dx) = dy? + 2ye* - dr = 0 (vi) (Qx2y + xy? + Bye + (ely? + 22y + Beldy = 0. Sol. (i) The given equation is xy sin 2x dr = (y? + cos® x) dy = 0 0) Comparing it with Mdx + Ndy =0 Here Mz ysin 2x, N =()? + cos?) Se a sin ds, Se (Bove sins) Si: aM aN ition (i) is exact. ince Wy Dar?“ equation (is Hence the solution is Max + f (terms in N not containing x) dy =¢ ye conatant or (i) The given equation is (x! — Oy + yd ~ (2°y — day? + sin yidy = 0 wai) Comparing it with Mdx + Ndy = 0 Here Maxt—2y24 4 . N =-(2x4y - day" + sin y) aM DS a sy+ yt ys BN nay + ay" Since = - x, equation (i) is exact. Hence the solution is Mass f (terms in N not containing x) dy = ¢ y constant, 150 GOLDEN DIFFERENTIAL EQUATIONS ie, J Gta aptsydes J -siny dy =e yconatant yo x x or Bey pty x ecosy sc or Be ty + co ya, 3 Gii) Please try yourself. [ame Sty eg? eat ae] * 2 = (iv) Please try yourself. Ans. ST tay' tet=c (v) The given equation is (2x2y 4 43 — 12xy? + By? — x0" + 6) dy + (12x2y + Dry? + Ad — Ay3 + Dyed — 09) dx = 0 oli) Comparing it with Mdx + Ndy = 0 Here M = 12r’y + 2xy? + dx3 = dy? + Qye™ = N= ely + ded — 1Dey? + By? — x0 + = 12x? + dey — 12y* + Be aM oy B a tey 4 1008-19980 + 22 . aM _ aN son cay Since Fy = Fe Caution (0 is exact. Hence the solution is Mae + | (terms in N not containing x) dy =¢ y- constant ie, J (Baty + Dey? + AP — dy? + Dye — eld + f Sy2dy =e y- snatant or dady + x2y? 4 xt — dry) + ye - 207 ty sc. (vi) Please try yourself. Cans, 4r4y3 + ax2y2 + 18xy =e] Example 5. Solve 2 + eee =0. (Welhi, 1997) Show that this differential equation represents a family of conics. Sol. The given equation is (ax + hy +g)dx + (hx + by +f dy = 0 wd Comparing it with Mdx + Ndy = 0 Here Msaxthyt+g,Neahz+by+f aM Since ye +. equation (i) is exact. Hence the solution is [Mae + { (terms in N not containing x) dy = c y-comstant SOLUTION OF DIFFERENTIAL EQUATIONS OF THE FIRST ORDER AND FIRST DEGREE 151 ie, J Gxthy rede + by + dy =e, > ecaant 2 x or a. S thy.reeeeb. + fyac, or ax! 4 Dhay + by? + Qe + 2fy —2e, =0 Replacing — 2c, by ¢ ax! + hay + By? + 2gx + 2fy +e =0 which evidently represents a family of conics. Another Form of Above Equation Show that the equation (ax + hy + g) dx + (hx + by + fidy =0 is exact and hence solve it. Example 6. Solve the following differential equations : ya? sy? 4a) & tafe? ey? a7) = 0 1 Gi) [>(++Z)omy] dx + (x + log x) (cos y -y sin y) dy = 0. Sol. (i) The given equation can be written as x(t? + y? = ahd + yc? +9? + a dy = 0 AD Comparing it with Mdx + Ndy =0 Here M = a(x? + 92-04); N =y (x? +9? + a?) aM oN Fy Oe Since x = x , equation (1) is exact. .. The solution is {Mas + J (terms in N not containing 2) dy =¢ a ekatant ie, J at ey—addes fy G2 +a%dy =e yocowtant or J be 4Q?-0xIdr+ [OF +a dy =e y-einwtant ‘ 2 4 2 or to? 09. i sat ane or (xt + 4 + 2x%y?) + 20%? — x2) = de a (2 +92? + 2a 2-22) =C, where C= 4c. (i) The given equation is 1 [>(1+ 2) a9] dx + (x + log x) (cosy — y sin y) dy = 0 152 GOLDEN DIFFERENTIAL EQUATIONS Comparing it with Mdx + Ndy = 0 Here May (142) cosy, N 20+ logs feos y— sin HF (124) tou y-ysingy, (142) (cosy -y sin» _ aM _ oN i jon i Since y 7 Or , the given equation is exact. «. The solution is Jo Max + J (Corms in N not containing x) dy =e y=eimatant 1 ie, J y(1+4) cos yar ee y~eonataat 1 or yoosyf [1+ Z]drmc or —ycosy (x + logad= ec, Example 7. Solve the following differential equations : dy | yoosx+siny+y . . @ Ge Sineszeosyrr 7? Gi) cos x + B dx + sinx dy =0 (ii) ye dx + (xe? + 2y) dy = 0 (iv) (5x! + 3x*y? = Qxy®) dx + (2x°y - 3x7y? ~ Sy4) dy = 0. Sol. (i) The given equation can be written as (y cos x + sin y + ylde + (sins +x 08 y + x) dy = 0 wa) Comparing it with Mdzx + Ndy =0 Here M=ycosx+ainy+y, N=sinx+xcosy +x aM an Sp Teme teosy +1, So =cose+ cosy +1 Since aS. the equation (1) is exact. Hence the solution of (1) is J) Max + [(terms in N not containing 2) dy =e y-crtant ie. J Geosx tein y+ 9dr =e p-euaant or ysinx+(siny+y)x=c. Gi) Please try yourself. (Ans. y sin x +x =e] ii) The given equation is ye dx + (xe + Byldy = 0 Comparing it with Maz +Ndy =0 Here Mayer, N= xe 42y SOLUTION OF DIFFERENTIAL EQUATIONS OF THE FIRST ORDER AND FIRST DEGREE 153 My Ws ieee eos BN ale.) +e sey + D . aM _ aN a ays Since So = SE the equation (1) is exact. Hence the solution of (1) is J Max J (terms in N not containing x) dy = ¢ mart ye ie, J ger dees f Qydy =e yn etastaat 2 or y Sat © or Mt ytae, {iv) Please try yourself. [Ans. x5 + x!y?- x4y3—95 = c] Example 8. Solve the following differential equations : GG) (eee x tan x tan y —e*) de + see x sec! y dy = 0 (ii) (2 xy 008 x? - xy + 1) dx + fin x? -i2) dy =0 (ii) (sin x cos y + €*) dx + (cos x sin y + tan y) dy =O (iv) [>(1+ 2) cory] artes logs —esiny) ay. Sol. (i) The given equation is (sec x tan x tan y—et) dx + sec.x sec? y dy = 0 we) Comparing it with Mdzx + Ndy =0 Here M =secx tan x tan y— et ,N = secx secty aM an By Teer tans wacky, Sp Tose x tan x sect y Since s x the equation (1) is exact. Hence the solution of (1) is J) Ms + (terms in N not containing x) dy = ¢ y-ecnstant ie, J (see x tan x tany—et)de=e ors tany secx—et =e. y-eonstant (ii) The given equation is (Qxy cos x2 ey + 1) dx + (ein 2? - x2)dy = 0 wf) Comparing it with Mdx + Ndy =0 Here M = 2xy cos x? 2xy +1, Nesinx?-2x? 154 GOLDEN DIFFERENTIAL EQUATIONS. aM ss 2rcosx?=2r , ay Since Oa 2%, the equation (1) is exact Hence the solution of (1) is J Mav [iterms in N not containing 2) dy = ponetant ie, J xy cos 28 Bry + ete = € y-enaant or yJ costdt-yf 2ede+zec — wheret=x? or ysint-yzt+xec or ysinx2=y4 rec (iti) Please try yourself [Ams,—cos xcos,y+ + e+ log eee y =} (iv) Please try yourself. [Ans. y (x + log x) +x cosy =e] Integrating Factors. Some of the equations which are not exact can sometimes be made exact after multiplying them by some suitable function of x and y. Such a function is called an integrating factor. ‘Thus, an integrating factor ofa differential equation is a factor such that if the equa- tion is multiplied by it, the resulting equation is exact. For example, consider the equation y dx —x dy = 0 (0) Here M-=yandN=-x Se aS , therefore the equation is not exact. yee ) Multiplying the equation by — , it becomes 22> ¥ x which is exact. (i) Multiplying the equation by » it becomes 222 =24¥ =0 or a(2)}=0 z which is exact. (ty Maltplying the equation by 23 becomes $= - 2 = 0 or dilog x logy) = 0 which is exact. ° s Sroge and J axe intgrating factors of). If a differential equation has one integrating factor, it has an infinite number of inte- &rating factors. LF. found by inspection. In a number of problems, a little analysis helps to find the integrating factor. The following differentials are useful in selecting a suitable integrating factor. SOLUTION OF DIFFERENTIAL EQUATIONS OF THE FIRST ORDER AND FIRST DEGREE 155 @yde + xdy = dexy) diy 524 a2) tin a2) (io) SF =a (tan) Y, +7 x xdy- ydx ) ., yde + xdy —d a ———— =d [I w) Oe [ive(2 (oi) FES ad flog oy) (ii) oe. a3 tog +37] witty = 3E ae a( ion 242) Example 1. Solve ydx ~xdy + Sx°y"o"" dx = 0. Sol. Since dxe"= dle"), the term Se4ye"" dx should not involve y?, This suggests that may be an LF. Multiplying throughout by Jy, we have 2" 24 saute de 0 y or a(2)seu’ ) = 0, which is exact. ¥, Integrating, we get ~ +e” =c which is the required solution. J Example 2. Solve xdy -ydr = rx? — y? de. dy — ydx Sol. The given equation is xdy — yd = x? Y ae or ta ade a (3) or asia Integrating, we get tint Zaxve or yexsin(k+0 7 ») = dx, which is exact. which is the required solution. Example 3. Solve adv + yey = 2 EB ¥ED) sy Sol. The given equation is xdx + yey - oPa (tant) =0 2 Integrating, we get aye tan] x ee or Sey? dettant® =C where C=20. * 156 GOLDEN DIFFERENTIAL EQUATIONS Example 4. Solve y (22y + e%) dx = e* dy (Lucknow, 1998) Sol. Re-writing the given equation, we have Qry? dx +y et dx -e* dy=0 Dividing by 5, aede + ed 0 oo aea+a(&]-0 e Integrating Beyee Example 5. Solve Beye aaa? Sol. Re-writing the given equation, we have addy +xy dx = 1-279? dx Dividing by x, xdy+ydz= fi-aty* = xdy+yde _ de or == or 4 (sin (gyn - =e fi-vy? = Integrating sin” (xy) -logx=c. RULES FOR FINDING INTEGRATING FACTORS Rule 1, If the equation Mdx + Ndy = 0 is homogeneous in x and y, ie., if M, N are 1 homogeneous functions of the same degree in x and y, then 74; yy i an integrat- ing factor. [Provided Mx + Ny #0] Proof. The given equation is Mdr + Ndy = 0 WD) where M, N are homogeneous functions of the same degree, say n, in x and y. «. By Euler's Theorem on Partial Differentiation, Bay Bm oe wali) and x aN +y aN saN . ar *” By Kedetne (1) 1 Multiplying () by » (Mr + Ny # 0), we get Mey NN Mx+Ny~ * Mr+Ny 2=° It will be exact if a M 2 N ay | Me+Ny| 7 ax | Me+Ny aM IN * aes Ny FF awa) (Me + Ny). Bn (mss sy) (Mx+ Ny)? (Mix + Ny)? ie, SOLUTION OF DIFFERENTIAL EQUATIONS OF THE FIRST ORDER AND FIRST DEGREE 157 aM aN oN aM or if Ny SM My 5, = Ms 5° -MN- NaS or if w(x Bhs y BM) at (2B oy BS) or if N.nM=M.aN [using Gi] which is true. Hence the result. Second Method. The given equation is Mdx + Ndy = 0 wf) where M, N are homogeneous functions of the same degree in x and y. #2) + (Mx - Ny) (£- *)] Now Made + Ney = 3 e+ =) Mde+Ndy = 2 [a + Ny) d(log xy) + (Mx— Ny) d (1 : Dividing by Mx + Ny (which is #0) Néx+Ndy 1 Mx- Ny sea 3 [aan Merny? Since M and N are homogeneous functions of the same degree in x vend .y, the expression ‘Mx- Ny Me+Ny . Maes Ney 1 * “ Merny 2 Mlos+ bs [=] | tons is homogeneous and equal to a function of - vsay {2 ). we Since ofa) *)=#(3) . Maes Ndy | 1 2 ” Merny 722 tbe) +3 F Joes 4 {tos which is an exact differential. Max +Ndy M = Me+Ny °° Me+Ny + wey o-° is an exact differential equation. Note. If the equation Md + Ndy = 0 is homogeneous in x and y and if Mx + Ny consists of only one term, use the method of integrating factor. Otherwise, proceed as in Type II by putting y = vx. Example. Solve : (i) ydz - 62 + y3) dy =0 (ii) (x2y — Bay?) dx ~ (x3 -Bx*y) dy = 0 (ii) (Bxy? — y°) dx ~ (2x"y — xy") dy = 0. Sol. (i) The given equations -x2ydx — (x* + ydy = 0 wal) Comparing it with Mdx + Ndy = 0 Mery; Ns-3-y¥ aM ‘s yy a2; =a? 158. GOLDEN DIFFERENTIAL EQUATIONS aM aN Since Sy ¥ Gy » equation (i) is not exact. But (i) is homogeneous in x and y 1 1 1 “ ego aoe Ma+Ny “ xBy-xty-y* Multiplying (é) by 5 — it becomes fa (22) aye0 , ~parlaty| oe Aid) which is exact. Sf .5. aM _oN_ 4 Hence the solution is x 1 x -Sae+[fdyec or = 25 + logyee. yen 2° J5 3y° (i) The given equation is (xy — Qxy*) dx — (<3 — Bx*y) dy = 0 =f) Comparing with Mdx + Nay = 0 Here M = x¥y—2ry?; N 2-29 4 Sxly aM _ oN ae = Bx? + Gay Since 2M «it equation (i) is not exact. ae ze But (é) is homogeneous in x and y . 1 ~~ (.— > - “ A HERS Sy -ay? rome) Multiplying ( by Sz a it becomes (ii) ; aM _aN_ 1 wih ere [> 3a] Hence the solution is 3 -=|de-[ -Ldy= ea ( ) j yore or 2 _alogx +3 logy =e. y (iii) Please try yourself. [ans Blog +2 ~2l0g y= SOLUTION OF DIFFERENTIAL EQUATIONS OF THE FIRST ORDER AND FIRST DEGREE Rule 2. If the equation Mdx + Ndy = 0 is of the form £,(xylydx + f,(xy)xdy = 0, then Wx-Ny is an integrating factor. (Provided Mx - Ny #0] wali) Proof, The given equation is Mdx + Nay = 0 where M=fooy N= fyoylx Now Mae +Nay= Hany) + a6-vv (*-“)] xy zy > Mdx + Ndy= ; [ate + Ny) d(log xy) + (Mx — Ny) d {ise 4) Dividing by Mx — Ny (which is #0) Mdr+Ndy 1 Fess yy z Me-Ny "2 |Mx-ny dilog wr a(tog ‘| 1) fay ay + frlayay 22) Aen + hawey 4 d 2 [see flay) xy 2088 = 3 [ra dilog xy) +d (ie | y Since ay =e" , flay) = f(e*”) = Flog xy) Max +Ndy 1 i 2 —_ =} Plog. dog.m) + 3 4 (log) which is an exact differential. Mas + Ney N = Mery 70 Sy v=o is an exact differential equation. = 1 ‘ ‘ :. 2 Hiz— iy 18 a2 integrating factor of Example. Solve (i) (xy? + 2x°y3)dx + (2y -xy?)dy = 0 Gi) (xy sin xy + cos xy)yde + (xy sin xy — cos xy) xdy = 0 (iit) (ay? + ay + Dydx + (x2y? ay + -Dndy = 0 (iv) (xy! + 229? + ayyd + (ty! — 2yF + ay edy = 0 () (1 + aylydx + 1 — yd = 0. Sol. (i) The given equation is (ay? + 2x°y8)dx + (22 - x'y?idy = 0 wf) Comparing it with Mdx + Ndy = 0 Here Me ay? + 2cty’, N=xty —aty? (Lucknow, 1997) aM 2, ON _ ayy _ gyaye OM a ay +632, a aay ay since MN uation (i) is not exact. ay” ax 160 GOLDEN DIFFERENTIAL EQUATIONS But (i) can be written as Gay + 229y7iydx + ay — xy7iady = 0 1 1 1 IF. =§ (= 35 yg a oe Mr-Ny x?y? ¢2r5y8—x2y? eaky? ax%y? » it becomes Multiplying ( by 3-5 12 11 ante +| > - =O (as ze (2 a\4 wpe OM aN which is exact, [: ay -ael Hence the solution is 1 & 1 Geyraleed yes ye comatant apa 1 or By) tg ea 3 logy =e, or vapsTle floes 1 —-—+2 - = &e, -— +2 ale =e. or wt log x - logy 1 OOF st log x — log y =e. (i) The given equation is Gy sin xy + cos zy)y de + (xy sin ay - cos xy) dy = 0 li) It is of the form flay) ydx + filxy) x dy = 0 Here Ms (xy sin ay + cos xyly N = (ay sin ay — c08 xy) x ee Mx-Ny — zy(xy sin xy + cos xy)~ xyxysin xy—cos zy) 2xy cos xy Multiplying (@) by 1, it becomes Bay cos xy [22= +p jes[E22- 2] dy =0 Zeosay * 3x | *| Zeon ay or (ytansy+2 2)ac+(xtanay-2) ay= 0 which is exact. [ OM A a eansy + sy sec? 39) Hence the solution is j (rtansy+4)ax+ J -3 dy =6, y=constant SOLUTION OF DIFFERENTIAL EQUATIONS OF THE FIRST ORDER AND: FIRST DEGREE 161 or 7 REE + Nog x logy =e, y or log see xy + log x —logy = ¢, or log sec xy + log ~ = loge or log 7 sec xy = loge y y or = seexy se or x 8eC XY = cy. y (iii) The given equation is (xy? + xy + Ly dr + (xy? xy + 1) xdy=0 i It is of the form f,(xyly dx + f,lxy)x dy = 0 Here Ma (xy? + ay + Dy Ne (a’y! ay + Die wet 1 _ 1 “S* = Mx-Ny we (x? y? + xy + Day - (x2? —xy+ Dey 2x?y? Multiplying ( by 2, it becomes axty 11 1 1a 1 (Sedesd)yeeo(t-sheghs)soes or (+2+ 2, )tee[s-2+ aro aM _ aN ich i , OM LON Ly which is exact. [ dy "be | Hence the solution is 1 1 1 yet dars | (-dJaree J ( y xy J ¥ or ye slogz + 2(-1) -iogy =e C5 or yetlogs-L-logysc or ay- slog 2 =e. xy xy yy (iv) The given equation is (xty* + xy? + xy)y dr + (ty —x2y? + wyixdy = 0 Ttis of the form f,ley)y dy + fay x dy = 0 Here Ms (xy! +2°y? + ayly, N= (ty4- 29? +e LF. = 1 1 1 Multiplying (i) by —— ay z ,it becomes 1 Eos deste} veer} Soh )se-0 Mx ~ wo «ty ty ay Gy = x8y papayas? 162 GOLDEN DIFFERENTIAL EQUATIONS v1 or [ots 2e sh Jare[sty- 2 Aap eo x xy yxy which is exact. Hence the solution is J (ot+te Jere J(-AJ)aree y=constant x xy y 2 x 1p. tbe e+ (2) tog y= or vy the + x} ~eyae 1 1 x or 5 xy?-—+lg=ec, 2 wey (v) Please try yourself. Rule 3. If in the equation Mdx + Ndy = 0 — ¢!f(4" js an integrating factor of (i) provided (ii) is true. SOLUTION OF DIFFERENTIAL EQUATIONS OF THE FIRST ORDER AND FIRST DEGREB 163 Example. Solve (i) Ge? +? + 2x) dx + Dydy = 0 (ii) (xy? = ) de —x2ydy = 0 2 ain (y+ = S)acet arma =0 72 4 ve (iv) (xy? + idx + 2fx*y? + x+y dy = 0. Sol. (i) The given equation is (x* + 9? + 2x) dr + 2ydy =0 uli) Here Maz xt 49? 42x,N=2y aM on Bn eo a _ aN ty de yO Ra net N 2y Multiplying (i) by e*, it becomes (ae + yet + 2xe*) de + 2yerdy = 0 [ are which is exact. Hence the solution is [tet + ye" + 210) de =e oes or (2 — Be 4 2) 4 yet + Bx - Dee or ef 2 Be 249? 4 Be -2) or eG@t+y ec (éi) The given equation is i [ote] =) 1 Here M=ay?-e ,N=-2xy aM an y Bey, Se Oy vse [ B-2-4 164. GOLDEN DIFFERENTIAL EQUATIONS Hence the solution is or Adi) -2 aed ot 3 1 1s “( se" des fe we ef Hence, from (ii), the solution is 2 a 1 at *3 et ne. (ii) The given equation is O+dst+ batdet dtm dy =0 fi) Meytiy+42 Nel ice aM 2 WN 1 dy Tt a eg at” OM _ aN ay a +y7-F+y)_ F4y) 3 fe) © Ve tay?) dati+y?) x 7! a ee el MOH IES eee ghee os Multiplying (i) by x, it becomes (y+ bys fades 1 rts cy2dy =0 which is exact. Hence the solution is J ytd hsb deee, ye constant 4 4 8 Byiy 2.18. or tg ep ee or Orly taxtyS 4x5 = 12c, or Baty + hy + x! xtyt 98 (iv) Please try yourself. Ans 22 +97+ me SOLUTION OF DIFFERENTIAL EQUATIONS OF THE FIRST ORDER AND FIRST DEGREE 165 Rule 4, If in the equation Mdx + Ndy =0 an _am a is a function of y only = fly) (say? then ¢/fld¢x is an integrating factor. Proof. The proof of Rule 4 is similar to the proof of Rule 3 and is, therefore, left as an exercise for the student. Example. Solve the following : i) (4+ Byldx + (xy? + By! —dxldy =O (ii) (Bxty! + Oey) de + Bxty?- dy = 0 (Gi) (Qxy4e? + 2xy? + ydx + O2y'e? -x*y? - 3x) dy = 0. Sol. (i) The given equation is (y* + 2y) de + (xy + 2y4 - Aeidy = 0 wi) Here Mey! + 2y,N =xy8 + 2y4- de 3M aN years aN _ aM ac dy _ by -4)-(4y9 +2) -874+2) 8 Sr na ay M yit2y yy Dd ¥ Syovey — J -3 stony — ey? 1 LR. = E408 we se SHE EMT” ye OG Multiplying @ by $ , it becomes (>>) aee(« 42y- Jarno which is exact. [: AN ‘| » Hence the solution is 2 2 = hx = 2 Pec. J (»+3) +f 2ydy =e or (+B) +9 ©. y~einstant (ii) Please try yourself, [Ans. 233 + 2? = cy] (ii) The given equation is (2xy‘e’ + 2xy* +») dx + Gdy'e’ — x2y?— 8x) dy = 0 w=) Here M = 2ryfe¥ + 2ryF +y, N= xtyte’ = x2? = 3x BM sche 4 sy7e) 4 6 + 1= Dayle + Bye +608 +1 WN AM gga ag? 4-4 age say ens (4) 166 * . GOLDEN DIFFERENTIAL EQUATIONS 4 Fr. edrow _f 7” _g-4egy — glory -yt “+ 1 Multiplying (1) by 5, we get Qe 1 2 [wre Fop)eee(ee oes which is exact. Hence the solution is 2 J (ase? + 2+ J) aree oer tee. y~comatant Rule 5. An equation of the form xty? (nydx + mxdy) + x*y! (pydx + qxdy) = 0 where a, b, c, d, m,n, p, q are all constants, has an integrating factor x*y®, where o, B are so chosen that after multiplying by x“ y®, the equation becomes exact. Note. a and fi can also be determined by solving the equations sn ateth btbet and ctarl 2 Othtl, qa ‘The following example wil liustrate the prooadure. Example. Solve the following : (i) 0? — 2yaPidx + (Bay? - xidy = 0 Gi) (2xy? + yidx — (xy - Oxidy = 0 Gil) (9? + 2x*y)dx + (2x? -xy)dy = 0 Civ) (2x*y - By Idx + (3x? + 2ry7dy = 0. Sol. (i) The given equation is (y3 — 2yx7)dx + (2xy? - x3)dy = 0 uD Here May) 2ya2, N= Bey? OM aN Sy nt -2t, = my? ae? “ x 2X (Gis nat exact. Now (é) can be written as *Y*ydx + Qady) + 2299 (— Byds — dy) = 0 which is ofthe form —-x*y’ (mydx + nxdy) + x°y? (pydx + qxdy) = 0 Let xty® be an intergrating factor of (i) Multiplying (i) by x", it becomes CatyP™— De2yP 1) ae 4 (Betty? 284990) dy 9 It will be exact if 2 (atyBed _p.gaet yet 2 (2a %*HyB _ 9358) SOLUTION OF DIFFERENTIAL EQUATIONS OF THE FIRST ORDER AND FIRST DEGREE 167 ie, if (B+ Sxty'® 28 4 D2"? = 2 C04 Dx8yP? (a4 aye ty o if B+3=2%a+1) [equating co-effi. of x°y***) land -2(8+ D=-(a+3) [equating co—effi.of x°**y*} . 2a-B-1=0 or if and «-28+1=0 or if a@=1,B = 1. Hence xy is an integrating factor. Multiplying (é) by xy, we have Gay! — Sxty de + (2x2y9 - xty dy = 0 which is exact. [: BA yt —an%y| Hence the solution is a Big J Gy! 2dew ey or y!, 5-297. 7 wey y-einstant or styty? 23) = 2c (Note. The given equation can be written as ay? (yd + 2ndy) + xy? (= 2ycdx = addy) = 0 Here a=0,b=2,.m=l1,n=2 c=2,d=0,p=-2,q=-1 a+a+l _ b+p+1 a+] B+3 pais oe m n = T 3 or = =2a=-f-1=0 wl) crard d+Pel | at8 Bet gy ei co AZ) P q 7 7 Solving (1) and (2), = a=Be1) Gi) The given equation is (2x2y? + yidx — Ory - axidy = 0 wali) Here M= 2r4y? #y, N == (xy = 3x) am an Set GS ao arty 8 oN LON Gis notexact ay tae is . Now (i) can be written as ay (2ydx — xdy) + x°y (yd + Sxdy) = 0 which is of the form xy? (yds + nady) + x44 (pydx + qxdy) = 0 Let x*y? be an integrating factor of (é). Multiplying (i) by x°y®, it becomes (ax 92 fatty) de (y0*DyOHE _ getty dy =O

Das könnte Ihnen auch gefallen